Autumn 2011

Download as pdf or txt
Download as pdf or txt
You are on page 1of 159

Math 131, Lecture 1

Charles Staats
26 September 2011

1 Introduction
Loosely speaking, there are two sides to mathematics: the ideas and the tech-
nical skills. Most people who say that they hate math have probably gotten
hung up on the technical side. And it is an unfortunate fact that the technical
side cannot be done away with. However, the ideas are what make the technical
side interesting. Without them, no one would ever have discovered the technical
side, and certainly no one would care to study mathematics as their life’s work.
In this course, I will try to flavor the technical details with the ideas that
explain why people were thinking like this in the first place. Think of studying
mathematics like studying a map. One can simply sit down and try to memorize
all the rivers, lakes, and mountain ranges. Or one can imagine how an explorer
might travel, and bring the landforms to life. A river, for instance, becomes
at once an obstacle, a water source, and a highway. Some rivers you can wade
across; others are difficult enough that you may want to build a bridge. My goal
is to present the mathematical “landforms” with some kind of narrative about
how the first explorers might have seen them, and why they built the things
they did.
The situation with calculus is especially tricky. The basics of calculus, as
invented by Newton and Leibniz in the late 1600s, might be seen as “explor-
ing on top of the clouds.” There are plenty of interesting things to explore on
top of these clouds, but you can never be sure what’s under your feet. You
might step on a spot that looks solid, only to find yourself standing on air.
In the 1800s, mathematicians (most notably Cauchy and Weierstrass) built a
solid “foundation” to fix this problem, which is not so much a foundation as a
skyscraper. In this course, we will try to explore both the “castle in the clouds”
and the skyscraper—called analysis—that holds it up. Even if you have seen
some calculus before, you have almost certainly not seen much of the skyscraper.

2 Proofs
One of the key things that distinguishes mathematics from other disciplines is
the presence of logical proofs. In physics, you know that a ball will fall when
you release it because it has done so every time you released it in the past. But

1
in some sense, there is no absolute reason why it would have to keep behaving
in this fashion. One can imagine that gravity might suddenly stop working
tomorrow. √
In mathematics, we know that 2 is irrational because we can show, defini-
tively, that it could never be rational. And in fact, the√way we prove this is
called proof by contradiction: imagine a world in which 2 were rational, and
show that this world is contradictory; thus, it cannot exist.
The Fundamental Theorem of Arithmetic. Every natural number greater
than 1 can be written as the product of primes in a unique way, except for the
order of the factors.
For example, 45 = 3 · 3 · 5.
Claim. Let n be a natural number greater than 1. Then n and n2 have exactly
the same prime factors.
Proof. If
n = p1 · p2 · · · pk
is a factorization of n into primes p1 , . . . , pk , then

n2 = p1 p1 · p2 p2 · · · pk pk

is a factorization of n2 into primes. In both cases, the primes factors are


p1 , . . . , pk .
It is a standard fact that any rational number can be expressed in lowest
terms. That is, we can write it as a quotient p/q such that p and q have no
prime factors in common.

Theorem. Let n be a natural number. Then n is either a natural number or
an irrational number.

Proof. Assume, by√way of contradiction, that n is rational, but is not a natural
number. Writing n in lowest terms as
√ p
n= ,
q

where p and q are natural numbers with no prime factors in common. Since n
is not a natural number, q > 1.
Claim. p2 /q 2 is already in lowest terms.
We want to see that p2 and q 2 have no common prime factors. But the prime
factors of p2 are precisely the prime factors of p; likewise, the prime factors of
q 2 are precisely the prime factors of q. And since p/q is in lowest terms, we
know that p and q have no prime factors in common. This proves the claim.

2
Now, since q > 1 and both are positive, we know q 2 > 12 = 1. Hence,
p /q 2 is not a natural number (the denominator is bigger than 1). But since

2

n = p/q, we can square both sides to find that

p2
n= .
q2
Thus, n is not a natural number. Since we originally assumed it was, this is a
contradiction.

Corollary. 2 is irrational.

Proof. By the theorem above√ (with n = 2), 2 is either a natural number or
an irrational number. Since 2 is not a natural number, it is irrational.

Bonus Exercise. Show that 3 31 is irrational.

3
Math 131, Lecture 2
Charles Staats
28 September 2011

1 Analysis is about inequalities, not equations


Traditional mathematics is about equations—determining when two quantities
are equal. To “calculate” a quantity means, typically, to find an equal quantity
that is easier to work with. For instance, when we convert a fraction to a
decimal, we obtain the same number in a form that is easier to add to other
numbers.
However,
√ this notion breaks down when we try √ to deal with irrational num-
bers like 2. No matter how many √ digits of 2 we calculate, we will never

find a decimal number equal to 2. The most we can do is to √ approximate 2.
Thus, for instance, when we state that the first few digits of 2 are 1.414, we
are really stating that √
1.414 ≤ 2 ≤ 1.415;
since all quantities are positive, we can square them to obtain the equivalent
inequality
1.4142 ≤ 2 ≤ 1.4152 ,

a statement that can be tested without already “knowing” the value of 2.
When we want to deal with real numbers (and in particular, with irrational
numbers), we almost always end up dealing with inequalities and approxima-
tions rather than actual equations. Thus, we are going to spend some time
reviewing how exactly inequalities may be manipulated.

A word on things to come: the “skyscraper” of analysis is all about


inequalities. However, once we get to the “cloud castle” of calculus, we will
be back to caring mostly about equations. Thus, somehow, in the process
of climbing to the top of the skyscraper, the inequalities get translated back
into equalities. This is done using rules like the following:

Theorem. (to be proved later in the course) Let x be a real number. If we


want to show that x = 0, it suffices to show the following: for every positive
number ε,
|x| < ε.

1
Typically, when you see the symbol ε (Greek letter epsilon), you should
think “small positive number.” This is purely psychological: the statement
would be just as correct if you replaced every ε with a y. Nevertheless,
this “psychological” choice of variable can provide an important guide for
intuition. When you see a statement like the theorem above, you should
get the following idea:
“If we can do a good enough job of showing that x is really close
to zero, we’ll know that x is actually equal to zero.”

2 Rules for manipulating inequalities


If you read Section 0.2 of the textbook, you’ll see a lot of talk about “solving”
inequalities. The homework problems will use this term, so you’ll need to make
sure you understand what the authors mean by it. However, I prefer to think
of “manipulating” inequalities rather than “solving” them. For instance, if you
use the authors’ methods to “solve” the inequality

x2 < 2,

you’ll get something like √ √


− 2 < x < 2.

However, since 2 is hard to calculate, the initial inequality may be easier to
work with than the “solved” version.
Nevertheless, the basic tools are the same whether you want to “solve” in-
equalities or simply “manipulate” them. Unfortunately, these tools, i.e., rules
for manipulation, tend to be more about nitpicking than interesting ideas. I’ve
distributed a handout of rules that you should use for reference. Here are a few
“traps” you may be tempted to run into, if you’re used to solving equations
rather than inequalities:
• “I can multiply both sides by the same number.” ISSUE: You need to
check the sign first. If you’re multiplying by a positive number, you’re
fine. But if you’re multiplying by a negative number, you need to reverse
the direction of the inequality sign.

• “I can square both sides.” ISSUE: This only works if both sides are
positive.
• “If I have an inequality like (x−a)(x−b) < 0, where a product is compared
to zero, I can split it into the factors: x−a < 0, x−b < 0.” ISSUE: What
you can actually say in this particular case is that x − a and x − b have
opposite signs. In other words, one is positive and the other is negative.
Quadratic inequalities are more complicated than quadratic equations.

2
If there’s an “interesting idea” in manipulating inequalities, it’s this: in some sit-
uations (for instance, in quadratic inequalities), we divide into cases, connected
by words like and and or. At this point, we are not only doing algebraic ma-
nipulations. We are also playing around with the logical relationships among
the different inequalities. Here’s an example:

Example. (Example 3, Section 0.2 in text) Consider the inequality x2 −x <


6. Much as in the case of quadratic equations, we start out by making one
side zero and factoring the other side:
x2 − x < 6
x2 − x − 6 < 0 (subtract 6 from both sides)
(x − 3)(x + 2) < 0 (factor)
Now, this single inequality is equivalent to the statement that x − 3 and
x + 2 have opposite signs. In other words,

(x − 3 < 0) and (x + 2 > 0) or

(x − 3 > 0) and (x + 2 < 0)
We analyze these two cases separately.
Case 1:
x−3<0 and x+2>0
x<3 x > −2.
A shorthand for (x < 3 and x > −2) is
−2 < x < 3.
Case 2:
x−3>0 and x+2<0
x>3 x < −2.
There are no values of x such that x > 3 and x < −2. A shorthand for a
statement that is never true is 0 = 1.
Combining the two cases: We see that our initial inequality is equivalent
to the statement
−2 < x < 3 or 0 = 1.
In other words, it is true precisely when at least one of the following is
true:
(i) −2 < x < 3
(ii) 0 = 1
Since 0 = 1 is never true, it follows that x2 − x < 6 precisely when −2 <
x < 3.

3
3 Assignment due Friday, September 30
Read “A Bit of Logic” and “Quantifiers” on pp. 4–6.
Problem Set 0.1, numbers 45, 46, 63, and 64. Problems 45 and 46 will be graded
carefully.

Read pp. 8–9.


Problem Set 0.2, numbers 3, 4, and 12. Problems 4 and 12 will be graded
carefully. DO NOT use the quadratic formula on problem 12.
Bonus Exercise. Show that the following three conditions on x are equiva-
lent:

(i) x < 2.

(ii) x2 < 2 or x < 0.


(iii) There exists y such that (x < y and y 2 < 2).

4
Math 131, Lecture 3
Charles Staats
30 September 2011

1 Statements and conditions


A mathematical statement is either true or false. For instance, 0 < 1 is true,
while 0 = 1 is false.
We can build statements out of other statements using the logical operators
and, or, and not.
Statement In Words True or False?
(0 < 1) or (0 = 1) At least one of the two state- True
ments (0 < 1), (0 = 1) is true.
(0 < 1) and (0 = 1) Both of the statements (0 < 1), False
(0 = 1) are true.
not (0 < 1) The statement (0 < 1) is false. False
Sometimes a statement may involve a variable. The statement x < 1 is
either true or false, but we can’t tell which until someone tells us what x is.
This sort of statement might be called a condition on x.
Two conditions on x are equivalent if they hold for exactly the same values
of x. For instance,
 the condition x 6= 0 is equivalent to the condition (x >
0) or (x < 0) , since in both cases, the statement is true precisely when x is
nonzero. There are several ways to say this:
P (x) is equivalent to Q(x).
P (x) if and only if Q(x).
P (x) ⇐⇒ Q(x)
They all mean the same thing.


2 Functions: writing f instead of
[Note: the following history is partly fictional. But it could have happened this
way, and in my opinion, it’s a lot more interesting to think about it like this
than just to go through a dry “definition of a function.”]
For many centuries, algebra was, essentially, the study of formulas. Periodi-
cally, when dealing with formulas, people would pose a problem that could not

1
be solved using existing formulas. For instance, to the ancient Greeks, such a
problem was, “What is the side length of a square with area 2?” They knew
that the answer would be a solution to the equation x2 = 2. Unfortunately, this
presented a dilemma, since they had no formulas to solve such an equation.
There were, roughly speaking, two approaches to this dilemma. One ap-
proach, which was taken by Diophantus of Alexandria in the third century A.D.,
was to accept that certain equations have no solutions, and then try to deter-
mine which equations had solutions and which did not. Diophantus produced
some marvelous mathematics this way, and the sorts of questions he asked have
become important in many areas—for instance, in modern cryptography.
Unfortunately, Diophantus’ marvelous mathematics was little comfort to the
farmer who wanted to know how long his fence should be to get a square corral
with a given area. The other approach, which might have been more useful to
said farmer, was to say, “well, since we don’t have a formula for this, let’s invent
one—and then figure out how to calculate it.” Thus, the square root was born.
As the centuries progressed, mathematicians continued to add new notation
to their formulas—exponential, logarithm, sine and cosine, and others. But
eventually, this approach stopped working. In studying differential equations,
the variety of solutions became so great that it was wholly impractical to invent
a new notation for every type of solution. Thus, they started using the same
notation, f (x), for many different “formulas.” They might say something like,
“Let f be defined as the solution to the differential √ equation under considera-
tion,” and then proceed to use f as though it were . Later on, they might
use the same letter f for the solution to a different equation.
In mathematics, notation is usually just notation. But sometimes, a new
notation can lead to new insights. For instance, the symbol 0 was originally
introduced as a placeholder, so that one could write down numbers like 101.
But once the symbol was introduced, people began to realize that it made sense
to think of zero as a number—a conceptual breakthrough.
In the case at hand, mathematicians began to realize that they could study
the “set of all things that can be written as f .” In trying to understand what
these “things that can be written as f ” really were, they came up with the
following definition.
Definition. A function f is a rule that, given a number x, outputs a number
f (x).

Let’s consider the case of the square
√ root function f , defined by f (x) = x
(or, if you prefer, defined by f = ). We would like to define f as follows:
For each number x, the function f assigns to x that number y such
that y 2 = x.
Unfortunately, this definition has a couple problems:
• This definition is ambiguous. For instance, if x = 1, then f (x) could
be either 1 or −1. To resolve this ambiguity, we require that f (x) be
nonnegative.

2
• If x is negative, there is no number y such that y 2 = x; in this case, f (x)
is undefined.
To resolve these difficulties, we make the following, better definition:
For each nonnegative number x, the function f assigns to x the
unique nonnegative number y such that y 2 = x.
The second difficulty, in particular, illustrates an important fact: a function
may be defined on only some real numbers.
Definition. The domain of a function is the set of all numbers x such that
f (x) is defined.
Definition. Let f and g be functions. We say that f = g if f and g have the
same domain, and for every value of x in that domain, f (x) = g(x).
Warning. If f is a function, it may be tempting to write something like

f = x2 + 1.

This “equation” makes no sense. f is a function, whereas x2 + 1 is a number


(even if we’re not sure which number it is). It does not make any sense to
ask whether a function is equal to a number; they are simply different kinds of
objects. If you use this sort of sloppy notation on homework or tests, you will
lose points for it.

3 Composing functions
In the functions above, I always used x for a variable. There is√nothing special
about x; the
√ square root function can be defined by f (t) = t just as easily
as f (x) = x. More importantly, we can plug in other things for a variable—
numbers, other variables, expressions, even other functions. For instance, if f
is defined by f (x) = x2 , then we may write things like

f (−2) = (−2)2 = 4
f (x + t) = (x + t)2 = x2 + 2tx + t2
f (x2 ) = (x2 )2 = x4 .

Note that none of these is a definition for f ; they are all consequences of the
definition that f (x) = x2 .
If f and g are both functions, then we may define a new function, denoted
f ◦ g, by
(f ◦ g)(x) = f (g(x)).
This is called the composition of f and g; it is read “f composed with g.”
Example. Let f be the function x 7→ x2 , and let g be the function x 7→ x2 + 1.
Compute f ◦ f , f ◦ g, and g ◦ f .

3
Solution. f ◦ f is defined by
(f ◦ f )(x) = f (x2 ) = (x2 )2 = x4 .
f ◦ g is defined by
(f ◦ g)(x) = f (x2 + 1) = (x2 + 1)2 = x4 + 2x2 + 1.
g ◦ f is defined by
(g ◦ f )(x) = g(x2 ) = (x2 )2 + 1 = x4 + 1.
We could just as well have computed g ◦ f by
(g ◦ f )(x) = (f (x))2 + 1 = (x2 )2 + 1 = x4 + 1.
Note that functional composition is not commutative: in the example above,
f ◦ g is not equal to g ◦ f .

4 Piecewise-defined functions
The easiest way to define a function is, of course, using an algebraic formula.
But in a way, this misses the whole point of functions—that they can be used
for rules that cannot be described using an algebraic formula. For instance, it
is a (not entirely obvious) fact that the “rule”
f (x) = the number y such that y 5 + 20y + 16 = x (1)
gives an unambiguous definition for a function f , and a (much less obvious) fact
that this function f cannot be expressed in terms of simpler algebraic functions
like rth roots. Even though we can’t “solve” the equation y 5 + 20y + 16 = x for
y in terms of x in the sense of giving a formula, one can show that the solution
exists as a function.
Another way to define functions is using a combination of logic and algebra.
For instance, the following are two perfectly good functions:
(
x3 − 7 if x < 1,
f (x) = 2
x if x ≥ 1.
(
−1 if x is rational,
g(x) =
1 if x is irrational.
You may have a tendency to think that functions like these are somehow less
“real” than functions defined using only algebra. You need to get past this.
Piecewise-defined functions, as they are called, are just as “real” as functions
given by algebraic formulae, and are extremely important. For instance, if you
want to actually compute approximate values for the function defined in (1)
above, your best bet may well be to construct a piecewise-defined function that
is very close to it, and compute the values of that function.
An extremely important piecewise-defined function is the absolute value
function.

4
Definition. The absolute value function is defined by
(
x if x ≥ 0,
f (x) =
−x if x < 0.

f (x) is typically denoted |x|, read “the absolute value of x.” Note that |x| is
always nonnegative.
Example. Let f be the function defined by
(
x+1 if x ≤ 1
f (x) =
x−1 if x > 1.

“Solve” the inequality


f (2x) ≤ x + 1.
Solution. Since f is piecewise-defined, we split into cases.
Case 1: 2x ≤ 1. In this case, the inequality reads 2x + 1 ≤ x + 1, so the case is
equivalent to

2x ≤ 1 and 2x + 1 ≤ x + 1
1
x≤ 2 and x≤0
x ≤ 0.

Case 2: 2x > 1. In this case, the inequality reads 2x − 1 ≤ x + 1, so the case is


equivalent to

2x > 1 and 2x − 1 ≤ x + 1
1
x> 2 and x−1≤1
1
x> 2 and x≤2
1
2 < x ≤ 2.

Combining the two cases, we see that the condition f (2x) ≤ x + 1 is equiv-
alent to the condition
x ≤ 0 or 21 < x ≤ 2.

5 Digression: Completing the square (avoiding


the quadratic formula)
The quadratic formula is, in my opinion, drastically overemphasized in most
algebra courses. It is rather ridiculous that people who have not studied math
in thirty years might walk around remembering some (probably wrong) variant
of “minus b plus or minus the square root of b squared minus four ac all over
two a” without any recollection of why this is significant. Thus, I am going to

5
forbid you to use the quadratic formula on anything you turn in (including tests
and homework). Instead, I will expect you to use the technique of completing
the square, which is a much more powerful idea that is in fact used to derive
the quadratic formula. It’s also easier to remember, in that the only formula
involved is (b/2)2 .

Example. (Example 13 in the book.) “Solve” the inequality x2 − 2x − 4 < 0.


Do not use the quadratic formula.
Solution. Recall the important process of completing the square: to complete
the square of x2 ± bx, add ( 12 b)2 . In our case, b = −2, so ( 21 b)2 = (−1)2 = 1.
So, we need to turn the left side into x2 − 2x + 1. We do this by adding 5 to
both sides.

x2 − 2x − 4 < 0
x2 − 2x + 1 < 5
(x − 1)2 < 5
p √
|x − 1| < |5| = 5

Thus,
√ √
− 5<x−1< 5
√ √
1 − 5 < x < 1 + 5.

6
6 Assignment: Due Monday, October 3
Section 0.2, problems 45 and 46. DO NOT use the quadratic formula, contrary
to the book’s instructions. Problem 46 will be graded carefully.

Skim Section 0.3 (pp. 16–22). Do the Concepts Review on p. 22 (answers on


p. 24) to see if you need to read the section more closely; don’t hand this in.
You may want to look at Example 3, p. 18. This process of completing the
square is important. Make sure you understand it.
Do Section 0.3, problems 17, 18, 23, and 24. Problems 18 and 24 will be graded
carefully.

Section 0.5, problem 2. This problem will be graded carefully.

7
Math 131, Lecture 4
Charles Staats
3 October 2011

1 Working with absolute values


Recall the absolute value function,
(
x if x ≥ 0,
|x| =
−x if x < 0.
When asked to “solve” an inequality or an equation involving absolute values,
it is always possible to get rid of the absolute values by splitting into cases.
However, this can be ridiculously involved. The first pair of absolute value signs
gives us two cases. If there is a second pair of absolute value signs, then each
of these two cases splits into two subcases, for a total of four subcases. If there
is a third occurrence of an absolute value, we end up with eight subsubcases.
And so on.
For this reason, we often try to take “shortcuts,” using rules for manipulating
absolute values.
The rules for multiplication and division are easy:
|ab| = |a||b|
a |a|
=
b |b|
If we want to do addition or subtraction, the rules are not nearly so nice. We
end up with inequalities rather than equations:
|a + b| ≤ |a| + |b|
|a − b| ≥ |a| − |b|.
The addition rule can be used to deduce the subtraction rule: If we set a = c
and b = d − c, the subtraction rule gives
|a + b| ≤ |a| + |b|
|c + (d − c)| ≤ |c| + |d − c|
|d| ≤ |c| + |d − c|
|d| − |c| ≤ |d − c|.
|d − c| ≥ |d| − |c|.

1
The textbook calls the addition rule the “Triangle Inequality.” This term
is properly reserved for another inequality. Consider three points P, Q, and R.
Let d(P, Q) denote the distance from P to Q.

The standard fact that “the shortest distance between any two points is a line”
tells us that
d(P, R) ≤ d(P, Q) + d(Q, R).
If a, b, and c are real numbers, they also represent points on the number line.
Moreover, the distance between a and b is precisely |b − a|, and so the triangle
inequality for absolute values is

|c − a| ≤ |b − a| + |c − b|.

We can deduce the addition rule from this: Let a = 0, b = α, and c = α + β.


These substitutions were chosen precisely so that

c−a=α+β
b−a=α
c − b = β.

Thus, the triangle inequality gives us

|α + β| ≤ |α| + |β|,

which is the addition rule.

2 Graphing functions
One of the keystones of modern mathematics is the interaction between algebra
and geometry via the graphing of equations. In some cases, one can use algebra
to prove a geometric result; you may have seen this sort of analysis used in
analyzing the conic sections. However, in this course, we will be going mostly in

2
the opposite direction: we will be using the geometry to gain additional insight
about the algebra. See, for instance, the discussion of the triangle inequality
above.
The basic approach to graphing functions is, of course, quite simple:
1. Choose some values of x.
2. Calculate and plot the points (x, f (x)).
3. “Connect the dots.”
Example. Graph the function f defined by f (x) = x2 .
Solution. We first calculate f at a few points:
x f(x)
-2 4
-1 1
0 0
1 1
2 4
Now, we plot these points and “connect the dots”:

And, in this case, it works like a charm!

Question: How do I know when I’ve plotted enough points?

You don’t—not really. Later on, we’ll discuss how to show definitively that
you’ve plotted enough points, but no one ever does this in real life. But here
are some general guidelines. They’re not guaranteed to work, but they usually
do if you’re smart about it.
1. Make sure it is “clear” how to connect the dots. If your points are too far
apart, either vertically or horizontally, you may need to plot some more.
Generally speaking, you want the graph to be going “up” or “down” for
several points at a time.

3
2. Use your knowledge of the function. If the graph you’ve drawn is a line,
then the function had better be equal to a function of the form f (x) =
mx + b; if it’s not, then you probably need to plot some more points.
If your function has an (x − a) in the denominator, then you are dividing
by zero at x = a. So, you probably want to plot extra points near x = a.

3. Test some extra points in between the one you’ve already plotted. When
you think you know what the graph looks like, plot a few more points in
between the ones you’ve already plotted. If they are about where your
drawing says they should be, that’s a good sign.
Question: How do I know I’ve got all the interesting features of the
graph?
The best answer to this is to use calculus. Since we can’t do that yet, it
may be helpful to try to figure out what the function looks like in the “boring”
part. Most of the functions we will give explicit formulas for this quarter will
look like axn for very positive and very negative values of x. When the function
starts looking like this, there’s a good chance you’re in the “boring” part.
You do probably want to make sure you get all the x-intercepts, i.e., all the
points where f (x) = 0.

Issue: Discontinuities; undefined points


You probably want to figure out what the function’s “natural domain” is,
i.e., where it is defined. Make sure to figure out what is going on at the “edges”
of this natural domain. If the domain can be written in interval notation, see
what’s going on near the (non-infinite) endpoints of all the intervals
If the function is piecewise-defined, you usually don’t want to try to “connect
the dots” between different pieces.

Things that can go right


For the most part, the discussions above focus on things that can go wrong.
Sometimes there are also things that can be helpful. For instance, lines are very
easy (more on this in a bit).
Other important techniques include translations. If you can write f (x) as
g(x) + c, where c is a constant, then the graph of f (x) can be obtained from
the graph of g(x) by translating up by c. This can be useful, because g might
be nicer algebraically than f . If you can write f (x) = g(x − c), then the graph
of f is obtained from the graph of g by translating g to the right by c.
Example. Graph the function f defined by f (x) = (x − 1)2 − 2.

Solution. If g(x) = x2 , then f (x) = g(x − 1) − 2. Thus, take the graph of g,


and translate it one to the right and down two.

4
Example. Recall the inequality from the end of the last lecture:

f (2x) ≤ x + 1,

where (
x+1 if x ≤ 1
f (x) =
x−1 if x > 1.
Graph the functions g(x) = f (2x) and h(x) = x + 1. Use the resulting graph to
study the set of values of x satisfying the inequality g(x) ≤ h(x).

5
3 Assignment 3 due Wednesday, October 5
Section 0.2, problems 53, 54, 57, and 63. Problems 54 and 63 will be graded
carefully.

Section 0.5, problem 13. This will be graded carefully.

Section 0.6, problems 13-16. Problems 14 and 16 will be graded carefully.

6
Math 131, Lecture 5
Charles Staats
5 October 2011

1 Quantifiers; rescuing a mess-up from last lec-


ture
Last lecture, I introduced the following two inequalities:

|a + b| ≤ |a| + |b| “Addition Rule”


|a − b| ≥ |a| − |b|. “Subtraction Rule”

I then did an somewhat abysmal job of explaining how the Addition Rule implies
the Subtraction Rule. I’m going to see if I can do any better the second time
around. I’ll also try to use this as an excuse to discuss quantifiers. Please let
me know immediately if I start talking gibberish again.
Example. Take the following statement as given: For every pair of real numbers
a and b,
|a + b| ≤ |a| + |b|.
Use it to prove the Subtraction Rule: For every pair of real numbers a and b,

|a − b| ≥ |a| − |b|.

Solution. The Addition Rule applies to every pair of real numbers. We’ve chosen
to write this pair as a and b. But if a and b are a pair of real numbers, so are b
and a − b. Applying the Addition Rule to the pair b, a − b, we obtain

|b + (a − b)| ≤ |b| + |a − b|
|a| ≤ |b| + |a − b|
|a| − |b| ≤ |a − b|
|a − b| ≥ |a| − |b|.

Let’s review the logic here. The Subtraction Rule has the appearance of a
condition on a and b: that |a + b| ≥ |a| − |b|. Let’s call this condition P (a, b).
Like all conditions, P (a, b) is either true or false, but in principle, we don’t know
which until someone tells us what a and b are.
However, we want to show that this condition P (a, b) holds for every possible
choice of a and b. Statements of the form

1
for all x, the condition P (x) holds
will be increasingly common as we progress into the study of limits, continuity,
and ultimately derivatives. The part of the statement “for all x” is called a
quantifier. It may seem more reasonable to talk about “the quantifier” when
we write the statement in symbols:

∀x, P (x),

where ∀ stands for “for all.” In this case, ∀ is the quantifier. The other important
quantifier is ∃, which stands for “there exists.” It showed up in one of the quiz
questions yesterday:

Let x be a positive real number. Show that there exists another


positive real number y such that y < x.
When you are asked to prove a statement involving quantifiers, there’s a
typical narrative structure that is involved. It’s easier to describe for the ∀
quantifier. If you are asked to prove that

for every positive real number x, P (x),


the proof typically starts out something like this:
Let x be a positive real number. We’ll show that P (x) is true.

An important note here is that when you say, “Let x be a . . . ,” you don’t get to
choose x. If it helps, imagine that someone else—an “opponent” or “enemy”—is
going to try to find an x to spite you. What you are doing for the rest of the
proof is showing that, no matter what x they choose, P (x) holds.
The narrative structure for a ∃ proof is a bit more confusing, because the
way you tell the proof is usually in the opposite order from the way you figure
out the proof. If you’re going to prove that

∃y such that y is irrational,

the proof you tell is probably going to have two steps:



1. Here’s a specific number y that I’ve dreamed up. For instance, y = 2.

2. Here’s why this specific y is irrational.


The trouble is, when you are figuring out the proof, it is often not clear what
y you should pick. You have to wrestle with the condition on y until you have
some y that you know (or at least suspect) works. And all of this initial work
gets left out of the story you tell.

2
2 Walking on clouds: What does this formula
mean?
At this point, I’m supposed to start motivating the notion of a “limit,” as we
ease toward the dreaded ε-δ definition. The problem is that, with the functions
we know how to use right now, there aren’t any really interesting or unexpected
limits—they tend to be a bit boring. Certainly, there is no real need for this
elaborate and potentially confusing definition. One of the few interesting exam-
ples is the difference quotient, not because it is hard to evaluate, but because
without understanding limits well, it is easy to be skeptical that the answer has
any meaning. One philosopher once called it the “ghost of a departed quantity.”
Suppose we are considering the function

f (x) = (x + 1)2 .

The graph of f is as follows:

(x2 , f (x2 ))

(x1 , f (x1 ))

Given any two distinct values x1 , x2 for x, there is a unique line through the
two points (x1 , f (x1 )) and (x2 , f (x2 )). We may calculate its slope as follows:

change in y
slope =
change in x
f (x2 ) − f (x2 )
=
x2 − x1
(x2 + 1)2 − (x1 + 1)2
= .
x2 − x1

3
Let’s see what happens when x1 = 0, so the first point is (0, f (0)) = (0, 1):

(x2 + 1)2 − 1
slope =
x2 − 0
x22 + 2x2 + 1 − 1
=
x2
2
x + 2x2
= 2
x2
x2 (x2 + 2)
=
x2
= x2 + 2

The interesting thing is that this formula can be evaluated when x2 = 0, even
though the original formula cannot. Moreover, if you look at the graph

you can get an idea that this is probably the slope of the tangent line at (0, 1).
This is something of a miracle, since the original reasoning for the formula
breaks down completely:
1. In the original formula, we’re dividing by zero. How can this possibly give
a meaningful answer?
2. If we look at how we derived the formula, it’s supposed to give us the
“slope of the line through (x1 , f (x1 )) and (x2 , f (x2 )).” But if (x2 , f (x2 ))
is the same point as (x1 , f (x1 )), this is ambiguous: there are infinitely
many lines through these “two” points, all having different slopes. So
where does this choice of a single line with slope 2 come from?

4
The original inventors of calculus more or less just accepted these miracles.
They used this formula to “walk on clouds,” even though they could not really
explain why dividing by zero could give a reasonable answer—or any answer at
all. And while they did great things this way, there were always mathematicians
who distrusted these “miracles.” Even more significantly, once people began
looking at arbitrary functions, they realized that these “miracles” don’t always
work; and it became necessary to understand this miracle so they could know
when it would work.
The solution to this turned out to be the definition of the limit, and was one
of the most important steps in building the “skyscraper” to support the “cloud
castle” of calculus.

5
3 Assignment 4 (due Friday, 7 October)
Section 0.2, problems 35, 36, 37, 38, 39, and 40. Problems 38 and 40 will be
graded carefully.

Solve each of the following inequalities two different ways:


(a) By factoring and then dividing into cases.

(b) By completing the sqare.


Make sure you get the same answer both ways.
1. x2 − 1 ≤ 0
2. x2 − 4x + 3 < 0
2
3. x + 2x − 3 ≥ 0
4. x2 + 2x − 3 > 0
2
5. x −x−6 ≤ 0
6. x2 − 3x − 28 ≥ 0
Problems 2 and 6 will be graded carefully.

Bonus Exercise. (Not due until Wednesday, 12 October; worth three points)
Find a piecewise-constant function f , defined on all x for which −2 < x < 2,
such that for all such x,
|f (x) − x2 | < 1.
Essentially, you are trying to approximate the squaring function by an easier-
to-calculate function.
Remember—you need to actually give a proof that your function f works.
This will involve showing, on each “piece” on which f (x) is a constant c, that
|c − x2 | < 1 on that “piece.” (You’ll never be able to find one c that works for
all x, which is why you need several “pieces.”)

6
Math 131, Lecture 6
Charles Staats
7 October 2011

1 Contrapositives in “real life”


Mathematics is often described as the “language of science.” But the truth
is, mathematics has its own language, based on informal1 logic. It is vital
to be fluent in this language in order to really understand mathematics. But
this language is not useful only in mathematics. A good understanding of the
language of informal logic is useful any time you want to debate anyone about
anything.
In mathematics, one of the crucial skills is to be able to move facilely among
different ways of saying the same thing. Very often, something that seems odd,
or at least non-obvious, when stated one way, will become obviously true (or
perhaps obviously false) when you move to a different way of saying them. Other
times, you can take a true statement, and understand it better by thinking about
different ways of saying it.
Right now, we will be concentrating on if-then statements. (These are some-
times called conditionals, but I’m going to avoid this term2 because it is so
similar to the term “condition” that I used for “conditions on x.”)
Consider the statement
Whatever does not kill you, makes you stronger.
At first glance, this does not look like an if-then statement. But we can make
it into one:
If something does not kill you, then it makes you stronger.
One advantage of an if-then statement is that we can take the contrapositive,
which is formally equivalent. Thus, the statement above is equivalent to

If something does not make you stronger, then it kills you.


Or, translating back into the original language,
1 Roughly speaking, “informal logic” uses words and sentences and is comprehensible to

humans. “Formal logic” uses symbols and is comprehensible to computers. The two-column
proofs you may have done in high school geometry fall somewhere in between.
2 Except for this one remark, of course.

1
Whatever does not make you stronger, kills you.
Here we’ve taken a statement that seemed reasonable, and showed that it means
the same thing as another statement that may seem absurd. This could be a neat
trick if you are in a debate with someone who is defending the first statement.
Now, suppose we’re trying to defend the first statement. In order to do that,
we have to think about the second statement in a way that makes it seem more
plausible. For one thing, we replace the too-general notion of “Everything” by
the more restricted notion of “every trial,” which is basically what we meant
anyway. Then, we do something like the following.
Every trial either kills you or makes you stronger. Thus, the only
way a trial can fail to make you stronger is by killing you.
One additional note: From a careful, logical perspective, the statement
“Whatever trial does not kill you, makes you stronger” does not exclude the
possibility that some trial might both kill you and make you stronger. We typ-
ically fill in this bit of information from common sense, and there is nothing
wrong with that in everyday language. But in mathematics, it is extremely
important to be able identify when we are filling in information from “common
sense” rather than logic, for two reasons. First, someone else’s “common sense”
might differ from ours, in which case we need to be able to defend our claims
with logic. Second, “common sense” is sometimes very wrong, as we will see
later in the course.

2 Saddling the Infinity beast


There is a great beast called infinity. This beast roams the plains of thought,
going places that most people can only wonder at. It has always been one of
the great mysteries. One of the greatest privileges of being a mathematician is
to be able to sit astride this beast, to go where it goes and see the wondrous
spectacles it sees.
Recall the quiz question that amounted to
Prove that there is no least positive real number.
Some people said something like the following:
The positive real numbers can be divided infinitely small, so there
is no smallest.
This is essentially the right idea, but the explanation is wrong. There are no
“infinitely small” numbers, so this notion of “dividing infinitely small” does not
make sense. Metaphorically, you’re headed in the right direction, but you’ve
spooked the Infinity beast, and it won’t carry you.
To hold yourself up, you need a saddle—something between you and the
Infinity beast that will keep you from falling off. One of the best saddles, and
the one that works here, is the notion of the arbitrary. You’d like to deal with

2
“infinitely small” numbers, but that does not work. Instead, you deal with
arbitrarily small numbers. Rather than trying to do things with infinitely small
numbers, you take a finite number, often called ε, and do something that works
“for arbitrarily small ε.” In other words, what you are doing will still work, no
matter how small ε gets. This “arbitrarily small” number ε is your “saddle,”
the “cushion” between you and the Infinity beast that lets you ride in comfort.
An important note here, worth repeating, is that you don’t get to choose ε.
If you choose a particular ε, whether it be ε = 1 or ε = .000001, you are limiting
how small ε can be. In order for ε to be arbitrarily small, and hence provide a
connection to the infinitesimal3 , you have to remember that ε is just, well, ε.
Generally speaking, it’s a small positive real number. You can’t pretend you
know more about it than that.
Coming back to the quiz question, no matter how small you make ε, ε/2
is always smaller. Thus, no matter how small you make ε, there is always
something smaller.

3 Order of quantifiers
Last lecture, I discussed a bit about the quantifiers “for all” and “there exists.”
Let’s consider how these are used in the statement of one of the quiz problems:
Suppose that x is a positive real number. Show that there is another
positive real number y such that y < x.
I mentioned in the last lecture that this includes the quantifier “there exists.”
But if you think about it, there are actually two quantifiers. The statement
could be re-written more symbolically as

∀x > 0, ∃y > 0 s.t. y < x,

where “s.t.” stands for “such that.” If you imagine playing a game against an
opponent, the opponent gets to move first (he gets to choose x), and then you
get to move, choosing y in response. As we’ve seen, no matter what x he chooses,
you can always choose y = 21 x and win.
Let’s see what happens if we reverse the order of the quantifiers and let you
move first:
∃y > 0 s.t. ∀x > 0, y < x.
In other words, “there exists a positive number less than all positive numbers.”
When you play this game, you lose: whatever y you choose, your opponent then
chooses x = y. Since x = y, it is not true that y < x, which is what you would
have needed in order to win.
This example illustrates something important about the order of quantifiers:
it’s easier to “win” if the ∃ quantifier comes last—in other words, if you move
3 “Infinite” means infinitely large. “Infinitesimal” means infinitely small. A woman who

confused these two once told a speaker, “I enjoyed your lecture very much and thought it was
of absolutely infinitesimal value.” This was not the compliment she apparently thought it was.

3
second, rather than first. If this seems somewhat counterintuitive, think about
it as a debate: it’s easier to win if you get the last word.
As a final point, let’s consider one formal definition of a limit:
Definition. Let f be a function whose domain includes all x > m, for some
finite m. We say that
lim f (x) = c
x→∞

if
∀ε > 0, ∃M > m such that if x > M, then |f (x) − c| < ε.
I don’t expect you to really understand this definition yet; we’ll go over it
more carefully next lecture, with more motivation. The key point to realize is
this: if you look at the quantifiers, you will notice that the ∃ quantifier comes
second. This means that in “playing the game,” you get the last word. This
does not guarantee that you will win, but it does give you a fighting chance.

4
Assignment 4 12 (due Monday, 10 October)
“Problems” 1 and 3 on page 33 of the book you will find at http://www.
phy.duke.edu/~rgb/Class/intro_physics_1/intro_physics_1.pdf. These
“problems” involve doing some reading—three times—and writing a couple of
short essays. This portion of the book gives advice on how to learn. It’s by one
of my favorite professors when I was a college student. (He was much better at
giving out candy than I am.) The essays will be collected and graded (by the
instructor).
One final note: The assignment will be, quite literally, impossible to complete
unless you start it by Friday, since part of the assignment is to work on it on
three different days.

5
Assignment 5 (due Wednesday, 12 October)
• Consider the following two problems from the quiz:
– Suppose that x is a positive real number. Show that there is another
positive real number y such that y < x.
– Either state the smallest positive real number, or prove that there is
no such thing.
Explain why these are really the same problem. This problem will be
graded carefully.
• For each of the following statements,
(a) Rewrite it as an if-then statement.
(b) Give the contrapositive. (You may use “bad” as an abbreviation for
“not good.”) Optionally, rewrite it in a form resembling the original,
rather than the “if-then” form.
Here are the statements:
1. Everyone with a beer has an ID.
2. Everyone on the plane has a ticket.
3. Every good boy does fine.4
4. Every good boy deserves fudge.
5. Good men die young.
6. Only good men die young. [Note: This one is tricky.]
Statements 2, 4, and 6 will be graded carefully.
• A friend of yours does not understand the formal definition of a limit.
Write a paragraph explaining it to him so that it makes sense. (You may
want to try this at least twice—once before the lecture on Monday, and
once afterwards. Ideally, you should turn in several versions that show
how your own understanding has improved.)
Bonus Exercise. (worth three points) Find a piecewise-constant function f ,
defined on all x for which −2 < x < 2, such that for all such x,
|f (x) − x2 | < 1.
Essentially, you are trying to approximate the squaring function by an easier-
to-calculate function.
Remember—you need to actually give a proof that your function f works.
This will involve showing, on each “piece” on which f (x) is a constant c, that
|c − x2 | < 1 on that “piece.” (You’ll never be able to find one c that works for
all x, which is why you need several “pieces.”)
4 This sentence is used as a memory aide in music theory for the sequence of letters EGBDF,

which partly explains why it is grammatically questionable.

6
Math 131, Lecture 7
Charles Staats
Monday, 10 October 2011

1 A bit of computer science


Suppose we have two computer programs for doing the same thing—say, multi-
plying matrices. You don’t need to know what this means, except that matrices
have size n. The bigger n is, the bigger the matrices are, and the longer it takes
to multiply them.
Let’s say the first computer program takes 100n3 operations to multiply two
matrices of size n, while the second computer program takes n4 operations to
multiply the same two matrices. If we want to multiply matrices of size n = 30,
let’s see how long it takes:
First program: 100n3 = 100 · 303 = 2,700,000
Second program: n4 = 304 = 810,000
Clearly, the first program is slower, taking 2.7 million operations instead of
810 thousand. On the other hand, speaking very loosely, a one gigahertz pro-
cessor can execute a billion operations per second.1 So, even the first program
will only take .0027 seconds. The second program is faster, but the first is so
fast already that no one will notice the difference.
On the other hand, suppose we want to multiply matrices of size n = 500:
First program: 100n3 = 100 · 5003 = 12,500,000,000
Second program: n4 = 5004 = 62,500,000,000
In this case, on our one gigahertz processor, the first program takes 12.5
seconds, while the second takes 62.5 seconds. Not only is the first program
faster in this case, but the times are long enough that we actually care. This
illustrates a general point about computer science:

• When a program takes f (n) operations to process something of size n, we


generally only care what happens when n is big. When n is small, the
program runs so quickly anyway that we don’t care.2
1 Please don’t tell any computer science professor I said this. I’m speaking very loosely.
2 This is a rule of thumb. There are plenty of exceptions.

1
Now, suppose you’re a computer scientist, and someone hands you two pro-
grams and asks you which is faster? You see that the first program takes f (n)
operations, and the second program takes g(n) operations. So, for any given
value of n, you can simply calculate f (n) and g(n) and see which is smaller
(hence faster).
The problem is, the person has told you absolutely nothing about what val-
ues of n they care about. Lacking this knowledge, you assume they probably
care most about what happens when n is very large. Thus, you might con-
sider the following criterion for telling them the f -program is faster than the
g-program:

• For all large n, f (n) < g(n). (1)

Unfortunately, it’s not clear what you mean by “large n.” One simple attempt
would be to say something like “n is large if n > 100.” This sort of idea would
allow you to produce more precise versions of (1):
(i) For all n > 100, f (n) < g(n).
(ii) For all n > 1,000, f (n) < g(n).
(iii) For all n > 10,000, f (n) < g(n).
Question. Which of these criteria is the hardest to prove / least likely to be
true? Which implies the others?
If you notice, each of these versions of (1) has the following form: you first fix
some N , and then take “n is large” to mean that “n > N .” You then get a
version of (1) that reads
• For all n > N , f (n) < g(n).
Unfortunately, you have no idea how fast the person’s computer is, so you have
no idea what N to choose. So why not let it be arbitrary?

• There exists N such that for all n > N , f (n) < g(n). (2)

If you recall our discussion of quantifiers as a “game,” you may notice that
we have just given ourselves an extra “move” by inserting a ∃ quantifier. Note
that you’ve given yourself the first move rather than the “last word.” You had
no choice: the opponent’s move of choosing an n > N does not even make sense
until you’ve told him what N is, so you have to go first.
Question. What happens if you give the opponent both moves?

2 Another saddle: the “sufficiently large”


If you think about it, we might have started out by asking, “What happens if n =
∞?” Unfortunately, asking whether f (∞) < g(∞) will spook the Infinity beast

2
so badly that we’re likely to get gored. Instead, in (2), we’ve just stumbled upon
another saddle that we can use to sit on the beast without actually touching it:
the “sufficiently large.” When a mathematician writes something like

For all sufficiently large n, P (n) holds,

she means that we get to choose what we mean by “n is large.” In symbols, the
statement above would be written

∃N s.t. ∀n > N, P (n).

3 How many times faster?


Let’s revisit computer science for a bit. Suppose we’re comparing two computer
programs for which the time required for an input of size n is, respectively,

f (n) = n + 1
g(n) = 2n.

It will turn out that for all n sufficiently large, f (n) < g(n).
Question. How exactly do we show this?

Thus, by our previous notion, the f -program is faster. However, suppose we


are asked the question, “How many times faster?” Let’s consider the following
table of values:
n 10 100 1,000 10,000
f (n)
0.55 0.505 0.5005 0.50005
g(n)

When n = 10, we see that the f -program takes 0.55 times as long as the g-
program. By the time we get to n = 10,000, the f -program takes 0.50005 times
as long as the g-program. It seems that the bigger n gets, the closer f (n)/g(n)
gets to one half. So, we might be tempted to say that “the value of the function
f /g at ∞ is 0.5,” and hence that in some sense, the f -program is twice as fast
as the g program.
Unfortunately, if we say it this way, we’ve spooked the Infinity beast. So,
let’s try out our new “saddle” of the ”sufficiently large”:
f (n) 1
• For all sufficiently large n, = .
g(n) 2
Unfortunately, this is simply not true. No matter how big we make n, f (n)/g(n)
will never be exactly equal to 0.5. What seems to be true is something more
like this:
• For all sufficiently large n, f (n)/g(n) is close to 1/2.

3
Again, the trouble is that we don’t know what exactly we mean by “close.”
Perhaps we mean that f (n)/g(n) is within .01, or .001, or .0001 of 1/2. But it
looks like it’s even closer than this.
f (n)
• For every possible notion of “close to 1/2,” for all sufficiently large n,
g(n)
is “close to 1/2.”
For an arbitrary (small) positive number ε, we get a notion of “close to 1/2” as
“within ε of 1/2.” Thus, in symbols, we may write the above as

f (n) 1
∀ε > 0, ∃N s.t. ∀n > N, − < ε.
g(n) 2

This leads us to the definition of a limit:


Definition. For a function h and a real number c, we say that c is the “limit
of h(n) as n → ∞,” written
c = lim h(n),
n→∞

if
∀ε > 0, ∃N s.t. ∀n > N, |h(n) − c| < ε.
This could also be rewritten as

For all ε > 0, there exists N such that if n > N, then |h(n) − c| < ε.

Writing it this way makes my claim last lecture that “you get the last word”
look a little bit better.

4
Assignment 5 (due Wednesday, 12 October)
• Consider the following two problems from the quiz:
– Suppose that x is a positive real number. Show that there is another
positive real number y such that y < x.
– Either state the smallest positive real number, or prove that there is
no such thing.
Explain why these are really the same problem. This problem will be
graded carefully.
• For each of the following statements,
(a) Rewrite it as an if-then statement.
(b) Give the contrapositive. (You may use “bad” as an abbreviation for
“not good.”) Optionally, rewrite it in a form resembling the original,
rather than the “if-then” form.
Here are the statements:
1. Everyone with a beer has an ID.
2. Everyone on the plane has a ticket.
3. Every good boy does fine.3
4. Every good boy deserves fudge.
5. Good men die young.
6. Only good men die young. [Note: This one is tricky.]
Statements 2, 4, and 6 will be graded carefully.
• A friend of yours does not understand the formal definition of a limit.
Write a paragraph explaining it to him so that it makes sense. (You may
want to try this at least twice—once before the lecture on Monday, and
once afterwards. Ideally, you should turn in several versions that show
how your own understanding has improved.)
Bonus Exercise. (worth three points) Find a piecewise-constant function f ,
defined on all x for which −2 < x < 2, such that for all such x,
|f (x) − x2 | < 1.
Essentially, you are trying to approximate the squaring function by an easier-
to-calculate function.
Remember—you need to actually give a proof that your function f works.
This will involve showing, on each “piece” on which f (x) is a constant c, that
|c − x2 | < 1 on that “piece.” (You’ll never be able to find one c that works for
all x, which is why you need several “pieces.”)
3 This sentence is used as a memory aide in music theory for the sequence of letters EGBDF,

which partly explains why it is grammatically questionable.

5
Assignment 6 (due Friday, 14 October)
Section 1.5, problems 1-9. Remember: It is not enough to get the right answer.
You have to convince the reader that your answer is right. Problems 2, 4, 6,
and 8 will be graded carefully.

6
Math 131, Lecture 8
Charles Staats
Wednesday, 12 October 2011

1 Recalling the definition of a limit


First, let’s recall the definition of a limit—both the informal and the formal
versions.

We say that lim f (x) = ` if


x→∞

Informal: For every version of “close to”, we can choose some meaning
for “large” such that if x is “large,” then f (x) is “close to” `.
Formal: For all real ε > 0, there exists N such that for all x > N ,

|f (x) − `| < ε.

The following table shows the correspondence between the informal version
and the formal version.

Informal Formal Explanation


For every version of For every ε > 0 Each ε gives us a meaning for
“close to” “close to”—namely, “within ε.”
we can choose some there exists N When we’ve chosen N , we say
meaning for “large” that “large” means “bigger than
N .”
such that if x is such that if x > N As we’ve said, x is “large” if x >
“large,” N.
then f (x) is “close then |f (x) − `| < ε. We’ve said “f (x) is close to `”
to” `. should mean that “f (x) is within
ε of `.” Now, |f (x) − `| is pre-
cisely the distance from f (x) to
`, so saying “f (x) is within ε
of `” is the same as saying that
“|f (x) − `| < ε.”

1
2 Newton’s “definition of a limit”
Consider the following statement from Isaac Newton’s seminal work, the Philosophiae
Naturalis Principia Mathematica:
Quantities, and the ratios of quantities, which in any finite time
converge continually to equality, and before the end of that time
approach nearer to each other than by any given difference, become
ultimately equal.
This was centuries before mathematicians came up with the correct definition
of a limit in order to build the “skyscraper” of analysis. Newton was trying to
build his “cloud castle” of Calculus. It’s kind of hard to see in the middle of a
cloud, so it’s no wonder he was confused: he thought he was proving a theorem
rather than stating a definition.
Nevertheless, this statement has some of the key aspects of the definition of
a limit. Newton understood that it is not enough just to say that one quantity
“approaches” another. He put in a key phrase: approaches nearer than by any
given difference. In other words, when we say that “f (t) approaches `,” we
really mean that f (t) becomes arbitrarily close to `. In more modern language,
Newton’s “difference” would probably be called ε. We would say that for any
given ε, f (t) must approach to within ε of `.
And he incorporated another key understanding—how exactly does this “be-
coming close” depend on t? Newton saw t as time. What we called f (t), he
might have called “the value of f once the time t has passed.” Letting t get
larger is, for him, simply letting a lot of time pass. And when we think about it
this way, we come to the following realization. In order for f (t) to approach `
“nearer than a given difference,” f (t) must become nearer than that difference
in finite time. In other words, there is a time N , after which f (t) becomes—and
remains—within ε of `.
Thus, in Newton’s language, we have the following definition of limit:
We say that a function f (t) (where t represents time) has a limit
` if for any given difference ε, within finite time, the quantity f (t)
approaches—and remains—nearer to ` than by ε.
Exercise. Relate this definition to the formal definition of a limit by making a
table like the one at the end of Section 1.

3 Computing limits when they exist


One of the interesting things about limits (as well as other major characters we
will meet in the study of Calculus) is that the usual methods of computing them
look practically nothing like the definition. The following “theorem” (it’s really
a bunch of theorems stated at the same time) is essentially copied from page 68
of the textbook, and is quite useful for evaluating limits. It gives situations in
which limits behave exactly as you might hope.

2
Theorem. (“Main Limit Theorem”) In the following equations, if the right
side makes sense, then the left side also makes sense and is equal to the right
side.

1. lim k = k
x→∞

1
2. lim =0
x→∞ x
h i h i
3. lim [f (x) + g(x)] = lim f (x) + lim g(x)
x→∞ x→∞ x→∞
h i h i
4. lim [f (x) − g(x)] = lim f (x) − lim g(x)
x→∞ x→∞ x→∞
h i h i
5. lim [f (x) · g(x)] = lim f (x) · lim g(x)
x→∞ x→∞ x→∞

f (x) lim f (x)


6. lim = x→∞
x→∞ g(x) lim g(x)
x→∞
h in
7. lim [f (x)]n = lim f (x)
x→∞ x→∞

“The right side makes sense” means, for now, that the limits in question
exist (as real numbers) and there is no division by 0.

This theorem can be proved from the definition of the limit. The proofs are
not even that difficult. But the only way they can ever be interesting is when
you do them yourself. Watching someone else do them is terribly boring, so I’ll
skip the proofs—at least for now—and move straight to discussing how to use
the theorem to actually compute limits.
Warning. If you use this theorem (typically, repeated applications of this theo-
rem) to compute a limit, then you will have shown, in the process, that the limit
exists. However, if you try to apply this theorem, and end up with something
that makes no sense, you will not have shown that the original limit does not
exist.
Example. (Example 2, p. 78 in the textbook) Compute
x
lim .
x→∞ 1 + x2

In particular, show that it exists.


Solution. The most obvious thing to try here is to apply Rule 6, which would
tell us that
x limx→∞ x
lim = ,
x→∞ 1 + x2 limx→∞ 1 + x2

3
assuming that the righthand side makes sense. Unfortunately, the right hand
side does not make sense: the limits on the righthand side do not exist.1
A more successful way to solve this problem is to first divide both the top
and the bottom by the highest power of x that appears in the denominator.

x x 1/x2
lim 2
= lim 2
· (algebra) (1)
x→∞ 1 + x x→∞ 1 + x 1/x2
1
x
= lim (algebra) (2)
x→∞ 12 +1
x
lim 1
x→∞ x
= (Rule 6) (3)
lim [( 1 )2 + 1]
x→∞ x
lim 1
x→∞ x
= 2 (Rules 3, 7) (4)
lim x1 + lim 1
x→∞ x→∞
0
= (Rules 2, 1) (5)
02 + 1
= 0. (6)

To the right of each line is written the justification: why do we know it is equal
to the previous line (assuming it is defined)?
A few words should be said on how we actually know the limits exist. If
we actually want to be careful here, our knowledge of the limits goes from the
bottom of the stack of formulas to the top. Because line (5) makes sense, the
theorem tells us that line (4) makes sense and is equal to it. Because line (4)
makes sense, the theorem tells us that line (3) makes sense and is equal to it.
And so on, all the way up to the top (which is what we cared about to begin
with).
General procedure for computing limits of rational functions:
A rational function, as you may recall, is a function of the form

an xn + an−1 xn−1 + · · · + a1 x + a0
f (x) = .
bk xk + bk−1 xk−1 + · · · + b1 x + b0

When faced with a function like this and asked to compute limx→∞ f (x), here
is a procedure that often works:
1. Multiply the numerator and denominator both by 1/xk .
2. Use the rules of the “Main Limit Theorem” to “distribute” the limit signs.
Bring them further and further “inside” the formula, until all the limits
are of the form limx→∞ 1/x = 0 or limx→∞ k = k.

1 In a more sophisticated point of view that we will adopt later, the numerator and the

denominator are both ∞. But ∞/∞ still does not make sense, as we will discuss.

4
Assignment 6 (due Friday, 14 October)
Section 1.5, problems 1-9. Remember: It is not enough to get the right answer.
You have to convince the reader that your answer is right. Problems 2, 4, 6,
and 8 will be graded carefully.

Assignment 7 (due Monday, 17 October)


Do the exercise at the end of Lecture 8, Section 2 on Newton’s “definition of a
limit.”

In the textbook, Section 1.5, Problems 15, 16, and 18. Problems 16 and 18 will
be graded carefully.

Complete the attached worksheet on graphing piecewise-defined functions.

5
Math 131, Lecture 9
Charles Staats
Friday, 14 October 2011

1 Remembering the definition of the limit


[Note: In case you have not figured it out yet, the definition of the limit is very
important, and it will be on the test.]
Recall the definition of the limit:

Definition. For a function f and a fixed number `, we say that

lim f (x) = `
x→∞

if
∀ε > 0, ∃N s.t. if x > N, then |f (x) − `| < ε.
Less formally:

For arbitrarily small positive ε, for sufficiently large x, f (x) is within ε of `.

This definition involves using two different “saddles” for the Infinity beast:
the saddle of the “arbitrarily small,” and the saddle of the “sufficiently large.”
It is the interaction of these two “saddles” that makes the definition of the limit
so intricate. In order to get the definition right, you have to get both of them,
and they have to be in the right order.
It may help to think of the “quantifier game.” In this definition, there are
three moves:
1. First, your opponent moves. He gets to choose ε. In other words, he gets
to choose how small is “arbitrarily small.”
2. Then, you get to move, by choosing N . Informally, you get to say how
large is “sufficiently large.” Your choice can depend on ε, which your
opponent has already chosen. But it cannot depend on x, which has not
been chosen yet.

1
3. Finally, the judge gets to go—to decide who wins. In a sense, he gets to
choose x; then, he determines whether this x does what you said (in which
case you win) or not.
To reiterate:

For arbitrarily small positive ε, for sufficiently large x, f (x) is within ε of `.


| {z } | {z } | {z }
opponent’s move your move judge’s decision

More formally,

∀ε > 0, ∃N s.t. if x > N , then |f (x) − `| < ε.


| {z } |{z} | {z }
opponent’s your judge’s decision
move move

2 Limits as x → c, where c is a real number


rather than ∞
Consider the function f defined by
x−2
f (x) = .
x−2
This function is not defined at x = 2, since computing it at that point would
involve dividing by zero. However, for every value of x other than 2, f (x) = 1.
Thus, the graph of f looks like this:

f (x)

2
There is a “missing point” at x = 2, but it is clear what the value f (2) ought
to be. Mathematicians have a way of stating, precisely, that “the value of f (x)
at 2 ought to be 1”: we write

lim f (x) = 1.
x→2

More generally, for a given number c and a given value `, we can claim that “as
x → c, f (x) → `,” which is also written

lim f (x) = `.
x→c

Note: When we write this notation, c is a value of x, whereas ` is a value of y.


In particular, in our case,

c=2
` = 1.

The definition of this sort of limit is quite similar to the definition we’ve
already encountered. For the previous definition, we needed the concept of
“arbitrarily large x” to deal let us talk about what happens at ∞ without
spooking the Infinity beast. In a sense, this is the same thing as “arbitrarily
close to ∞.”
This time around, we’re trying to understand what is (or should be) happen-
ing at c without actually touching c. (If you like, we want to avoid “spooking
the c-beast.”) The notion that replaces “arbitrarily large” is “arbitrarily close
to c.” Thus, we get the following definition.

Definition. We say
lim f (x) = `
x→c

if

For arbitrarily small ε > 0, when x is sufficiently close to c, f (x) is within ε of `.


| {z } | {z } | {z }
opponent’s move your move judge’s decision

More formally,

∀ε > 0, ∃δ > 0 such that if |x − c| < δ and x 6= c, then |f (x) − `| < ε.


| {z } | {z } | {z }
opponent’s your judge’s decision
move move

The condition x 6= c is added since we want to figure out what the value
“should be” at c, without actually touching c.

3
Example. Consider the function f defined by
(
2x − 1 if x 6= 2,
f (x) =
4 if x = 2.

Its graph looks like this:

f (x)

Let’s use the ε-δ definition of the limit to show that

lim f (x) = 3.
x→2

Note: When looking at this sort of example, we are not using the formal
definition of the limit to better understand the function f . We are using the
function f to better understand the definition. The formal definition becomes
really useful when we are dealing with functions f for which we don’t have
formulas.
Solution. Let ε > 0 be given (by our opponent; we can’t choose it). Before we
go around choosing δ haphazardly to set what is “sufficiently close to 2,” let’s
anticipate what the judge will say. In other words, let’s “solve” the inequality
he cares about as best we can, without knowing ε:

|f (x) − 3| < ε.

Since the judge does not care what happens when x = c = 2, we can assume

4
f (x) = 2x − 1. In this case, the judge’s “test” is whether

|(2x − 1) − 3| < ε
|2x − 4| < ε
2|x − 2| < ε
|x − 2| < 21 ε.

Now, we could proceed to finish “solving” the inequality; but in this case, that
would be counterproductive. The condition we impose, by our choice of δ, is
that
|x − 2| < δ.
Thus, if we set δ = 12 ε, we are guaranteed that the judge will like all the
“sufficiently small” values of x we allow him to look at.
Important: δ may depend on ε, and usually will. (Since our opponent has
already chosen ε, we’re allowed to use it.) But δ cannot depend on x. (The
judge doesn’t choose x until after we’ve already chosen δ.)

5
Assignment 7 (due Monday, 17 October)
Do the exercise at the end of Lecture 8, Section 2 on Newton’s “definition of a
limit.”

In the textbook, Section 1.5, Problems 15, 16, and 18. Problems 16 and 18 will
be graded carefully.

Complete the worksheet on graphing piecewise-defined functions.

Test Wednesday, 19 October


The test includes lectures through Wednesday, October 12, and assignments 1
through 7 (but not assignment 4.5). Note: The assignment numbers are one off
from the lecture numbers, since I gave no (non-bonus) assignment on the first
day of class. In particular, although the quiz does not include any new material
from today’s lecture, it does include the homework set due Monday.

Anything that appeared on a quiz will probably show up in some form on the
test. (Exception: no contrapositive questions.) Anything that appeared on a
non-bonus homework question might show up on the test.

If I said something in a lecture that did not make it in any form into a quiz or
homework question, then it will not be on the test.

6
Math 131, Lecture 10
Charles Staats
Monday, 17 October 2011

1 Definition: Limits as x → c
Recall from last time the definition of the limit of f (x) as x → c:

Definition. We say
lim f (x) = `
x→c

if

For arbitrarily small ε > 0, when x is sufficiently close to c, f (x) is within ε of `.


| {z } | {z } | {z }
opponent’s move our move judge’s decision

More formally,

∀ε > 0, ∃δ > 0 such that if |x − c| < δ and x 6= c, then |f (x) − `| < ε.


| {z } | {z } | {z }
opponent’s our judge’s decision
move move

A couple of notes on this definition:


• When we wanted to compute limx→∞ f (x), this depended only on the
function f . If we want to compute limx→c f (x), we will, quite probably,
have a different limit for every different choice of c.

• We specifically exclude the “judge” from looking at the value of f (x) when
x = c, because the limit is supposed to detect what “should be” going on
at c without actually touching c. This was not an issue for limx→∞ f (x),
where we’re sort of “setting c = ∞,” because f (∞) does not make sense
anyway.

1
2 Examples: Using the ε-δ definition
At this point, we’ll begin trying to understand the definition better by doing
some examples of ε-δ proofs. We are doing this to help us understand the defini-
tion, and the concept, of limit, which is much more useful in more complicated
situations.
Example. Consider the function f defined by
(
2x − 1 if x 6= 2,
f (x) =
4 if x = 2.

Its graph looks like this:


f (x)

Let’s use the ε-δ definition of the limit to show that


lim f (x) = 3.
x→2

Note: When looking at this sort of example, we are not using the formal
definition of the limit to better understand the function f . We are using the
function f to better understand the definition. The formal definition becomes
really useful when we are dealing with functions f for which we don’t have
formulas.
Let ε > 0 be given (by our opponent; we can’t choose it). Before we go
around choosing δ haphazardly to set what is “sufficiently close to 2,” let’s
anticipate what the judge will say. In other words, let’s “solve” the inequality
he cares about as best we can, without knowing ε:
|f (x) − 3| < ε.

2
Since the judge does not care what happens when x = c = 2, we can assume
f (x) = 2x − 1. In this case, the judge’s “test” is whether

|(2x − 1) − 3| < ε
|2x − 4| < ε
2|x − 2| < ε
|x − 2| < 21 ε.

Now, we could proceed to finish “solving” the inequality; but in this case, that
would be counterproductive. The condition we impose, by our choice of δ, is
that
|x − 2| < δ.
Thus, if we set δ = 12 ε, we are guaranteed that the judge will like all the
“sufficiently small” values of x we allow him to look at.

Important: δ may depend on ε, and usually will. (Since our opponent has
already chosen ε, we’re allowed to use it.) But δ cannot depend on x. (The
judge doesn’t choose x until after we’ve already chosen δ.)

If you recall the discussion of the “narrative” of a proof with quantifiers, the
way you “tell” a proof is often quite different from the way you work it out.
Now that we’ve worked out what the value of ε should be, let’s “tell” the story
of what goes on in the courtroom—without trying to get into the characters’
heads (as we were, earlier, by anticipating the judge). Just the facts, ma’am.
Solution. Let ε > 0 be given. Set δ = 21 ε. Assume |x − 2| < δ and x 6= 2.
Since x 6= 2, we know f (x) = 2x − 1. Hence,

|f (x) − 3| = |2x − 1 − 3|
= |2x − 4|
= 2|x − 2|
< 2δ
= 2( 21 ε)
= ε.

Thus, under these hypotheses, |f (x) − 3| < ε, as desired.


Hence, lim f (x) = 3.
x→2

3
Test Wednesday, 19 October
The test includes lectures through Wednesday, October 12, and assignments 1
through 7 (but not assignment 4.5). Note: The assignment numbers are one off
from the lecture numbers, since I gave no (non-bonus) assignment on the first
day of class. In particular, although the quiz does not include any new material
from the Lectures 9 and 10, it does include the homework set due Monday, 17
October.

Anything that appeared on a quiz will probably show up in some form on the
test. (Exception: no contrapositive questions.) Anything that appeared on a
non-bonus homework question might show up on the test.

If I said something in a lecture that did not make it in any form into a quiz or
homework question, then it will not be on the test.

Assignment 8 (due Friday, 21 October)


Give ε-δ proofs of the following facts:

lim 7x = 0 (1)
x→0
lim 2x = 2 (2)
x→1
lim 4x + 1 = −1 (3)
x→− 12

lim 1 x − 2 = − 12 (4)
x→5 2

They will all be graded carefully.

4
Math 131, Lecture 11
Charles Staats
Friday, 21 October 2011

1 Some notes on the test


First, some of you may be surprised, when you get back your test, to see how
many points I may deduct even when you get the “correct answer.” For instance,
on the first problem, I gave a number of people scores of 4/10, even though they
had the right “answer.”
The way I see it is this: When I ask you a question, I’m not just asking you,
“Where is London?” I’m asking you, “How do I get to London?” Someone who
tells me to get to London by walking across the Atlantic Ocean, even though
they have the right “answer” (London), will receive fewer points than someone
who tells me to take a boat to Madrid. At least the second person will get me
to the right continent without drowning.
Here are some common errors that were made on the test:

• The condition on x given by, for instance,

1 < x and x<2

can be abbreviated as
1 < x < 2.
The condition
1 < x or x<2
has no such abbreviation. These sorts of “combined statements” can only
be used for and. If you are dealing with an or statement, you have to
write it out in full, with the two statements included.
[Incidentally, the particular or statement above is actually equivalent to
the statement x = x. Why?]
• |2x + 7| > 5 is an or condition. |2x + 7| < 5 is an and condition.

• When you complete the square on 2(b), you should end up with

(x + 2)2 ≥ 25.

1
At this point, since the right side is positive, you take the square root of
both sides, getting
|x + 2| ≥ 5.
You can then the rules for solving absolute value inequalities.
If you were solving an equation, you would probably want to use a ± sign
rather than an absolute value. This does not work reliably for inequalities.
• It is certainly possible for f (x) to equal its limit for large x. We just
don’t often look at such examples because they are not very interesting.
However, when we study limits as x → c, we do explicitly disregard what
happens at x = c. It’s important to distinguish between what happens
to the y-values (f (x) can equal c, although it does not have to) and the
x-values (x cannot equal see—at least, as far as the judge is concerned).

• When we write either lim f (x) = `, the number ` is a value of y, not a


x→∞
value of x:

The same holds for the statement lim f (x) = `.


x→c

2 Limits as x → c: an intuitive picture


It should come as a surprise to no one that a few minutes of the lecture today
will be spent on the definition of the limit. Let’s take a moment for a terribly
imprecise, but intuitively useful, version:

2
Definition. (Terribly Imprecise Version) We say that

lim f (x) = `
x→c

if the following holds:


When x is close to c, then f (x) is close to `.

Here’s the picture:


y

This relates to the more precise definitions as follows:


1. First, the opponent decides what it means to say “f (x) is close to `.” He
does this by choosing ε > 0, and then saying “f (x) is close to `” means
precisely “f (x) is within ε of `.”
2. Second, we, knowing what ε the opponent has chosen, get to decide what
it means to say “x is close to c.” We do this by choosing δ > 0, and then
saying “x is close to c” means precisely “x is within δ of c (but not equal
to c).”
3. Finally, the judge takes our definitions and decides whether or not the
basic statement is true: “When x is close to c, then f (x) is close to `.” If
it is true, we win; if not, the opponent wins.

3
Assignment 9 (due Monday, 24 October, 2011)
Read Section 1.1. If you aren’t comfortable with sine and cosine, don’t pay too
much attention to the examples involving them. You do need to understand the
picture in Example 6, however.

Section 1.1, Problems 29 and 30. These problems are (unusually) of the “answers
only” variety. Problem 30 will be graded carefully.

Section 1.2, Problems 11–15. This time, the proof needs to be done carefully.
Problems 12, 14, and 15 will be graded carefully.

Assignment 10 (due Wednesday, 26 October, 2011)


Consider the piecewise-linear function f defined by


3x + 11 if x ≤ −3,
1
f (x) = 1 − 3 x if − 3 < x ≤ 3,


3−x if x > 3.

1. Graph this function. Carefully.


2. Use the graph to “guess” what lim f (x) and lim f (x) are. If it looks like
x→−3 x→3
the two-sided limits don’t exist, sleep on it, and then try graphing the
function again.
3. Give ε-δ proofs that the two limits in the question above are what you
say they are. Remember: You should have one proof for each (two-sided)
limit. (Hint: you will probably want to let δ = min{δ1 , δ2 }, for appropriate
values of δ1 and δ2 .)
The third problem will be graded carefully.

Section 1.2, Problems 1–6. Problems 2, 4, and 6 will be graded carefully.

Section 1.3, Problems 1, 2, 5, and 6. Be sure to follow the instructions—these


problems are about using the Main Limit Theorem carefully, not about finding
the limits (those are very easy to guess). Problems 2 and 6 will be graded
carefully.

4
Math 131, Lecture 12
Charles Staats
Monday, 24 October 2011

1 Ways limits can fail to exist


1.1 Jumps; one-sided limits
We consider the function f defined by
(
1−x if x < −1,
f (x) =
2+x if x > −1.

We have not defined this function at x = −1, but for the purpose of considering

lim f (x),
x→−1

f does not have to be defined at −1; and even if it is, we don’t care what its
value is.
f (x)

x
−4 −3 −2 −1 1 2 3 4
−1

−2

−3

−4

1
In this situation, the limit does not exist. To handle “jumps” like this, we have
the notion of one-sided limits.
Definition. We say that “f (x) approaches ` as x approaches c from the left,”
written
lim f (x) = `,
x→c−

if
∀ε > 0, ∃δ > 0 s.t. if c − δ < x < c, then |f (x) − `| < ε.
The boxed part says that “x is to the left of c and within δ of it”:

x
( )
c−δ c c+δ
δ δ

We say that “f (x) → ` as x → c from the right,” written

lim f (x) = `,
x→c+

if
∀ε > 0, ∃δ > 0 s.t. if c < x < c + δ , then |f (x) − `| < ε.

x
( )
c−δ c c+δ
δ

Exercise. Using this ε-δ definition, show, for the function f defined above, that

lim f (x) = 2
x→−1−

lim f (x) = 1.
x→−1+

Theorem. The two-sided limit limx→c f (x) exists if and only if both the one-
sided limits exist and are equal. In this case, we have

lim f (x) = lim f (x) = lim+ f (x).


x→c− x→c x→c

This theorem is not that difficult to prove, but we will refrain because of
time constraints. The basic idea is as follows: when the opponent gives us an
ε, we
• Find a δ1 that works for the left-hand limit.
• Find a δ2 that works for the right-hand.
• Set δ = min{δ1 , δ2 }.

2
1.2 Infinite limits
We say lim f (x) = ∞ if
x→c

Informal: For arbitrarily large K, when x is sufficiently close to c, then


f (x) > K.

Formal: ∀K, ∃δ > 0 s.t. if 0 < |x − c| < δ, then f (x) > K.

We say lim f (x) = −∞ if


x→c

Informal: For arbitarily negative K, when x is sufficiently close to c, then


f (x) < K.

Formal: ∀K, ∃δ > 0 s.t. if 0 < |x − c| < δ, then f (x) < K.


Example. If
1 1
f (x) = + ,
(x + 1)2 x−1
then
lim f (x) = ∞,
x→−1

while limx→1 f (x) does not exist in any sense. However,

lim f (x) = −∞
x→1−
lim f (x) = ∞.
x→1+

3
y

x
−4 −3 −2 −1 1 2 3 4

−1

−2

−3

−4

1.3 Limits that just plain don’t exist


Consider the function g defined by
(
0, if x is rational,
g(x) =
1, if x is irrational.

Consider
lim f (x).
x→0

No matter how small x is, there are always smaller values at which f is 0 (say,
x = n1 , for some really big integer n) and smaller values at which f is 1 (say,

x = n1 2, for an even bigger integer n). So, no version of the limit as x → 0
can exist—not the left-hand limit, not the right-hand limit, not even if we allow
limits that are ±∞.

4
y

x
−4 −3 −2 −1 1 2 3 4

−1

−2

−3

−4

2 ε-δ proofs for more complicated functions


Example. Let f be the function defined by

f (x) = x2 .

Show that
lim f (x) = 4.
x→2

First, we do a preliminary analysis. The definition of limit, applied in this


specific instance, is as follows:

∀ε > 0, ∃δ > 0 such that if 0 < |x − 2| < δ, then |x2 − 4| < ε.

Let’s look, specifically, at what the judge cares about:

|x2 − 4| < ε ⇐⇒ |x − 2||x + 2| < ε


ε
⇐⇒ |x − 2| < .
|x + 2|

5
At this point, it might be tempting to say, “Let δ be ε/|x + 2|.” However, to
set δ like this, we’d have to know x, and we don’t: the judge does not choose x
until after we’ve chosen δ.
This is the sort of thing that makes ε-δ proofs so much more difficult for
nonlinear functions: We have to control x to make two things happen at once:
• Make sure ε/|x + 2| does not get too small.
• Make sure |x − 2| does not get too big.
The second task is easy—it’s precisely what the δ is designed to do. But the
first task is much harder, and it’s where we have to start.
Suppose we know that |x − 2| < 1. If you “solve” this inequality, you get
precisely that 1 < x < 3. Thus, x + 2 is clearly positive, and so |x + 2| = x + 2.
Thus, we have

1<x <3
3<x+2 <5
3 < |x + 2| < 5
1 1 1
> >
3 |x + 2| 5
ε ε ε
> > .
3 |x + 2| 5
Note: in the inequalities above, each line implies the next, but is not necessarily
equivalent.
Thus, we have:
ε ε
If |x − 2| < 1, then < .
5 |x + 2|
As long as we choose δ ≤ 1, we have some control over how small ε/|x + 2| can
be. This takes care of the first, problematic task. Since we’ve done that, the
second task is comparatively easy: we need to ensure that
ε
|x − 2| < .
5
This works as long as δ ≤ ε/5.
So, to accomplish the first task, we need δ ≤ 1. Once we’ve done this, to
accomplish the second task, we need δ ≤ ε/5. Since 1 and ε/5 are both positive
numbers, we can simply set
n εo
δ = min 1, .
5
Now, we’ve finally got a plan. Let’s head into the courtroom and see what the
judge says.
Proof. Let ε > 0 be given. Set
n εo
δ = min 1, .
5

6
Now, suppose 0 < |x − 2| < δ. Then we have
|x2 − 4| = |x − 2|x + 2.
To proceed further, we need to know something about |x + 2|. Since δ ≤ 1, we
know
|x − 2| < 1
−1 < x − 2 < 1
1<x<3
3 < x + 2 < 5,

and consequently, x + 2 is positive.

3 < |x + 2| < 5.
Hence, |x + 2| is a positive number less than 5. Thus, we have
|x2 − 4| = |x + 2||x − 2|
< 5|x − 2|
< 5δ,

since |x − 2| < δ
ε
<5· ,
5
since δ ≤ ε/5

= ε.
If |x − 2| < δ, then |x2 − 4| < ε. Hence, the limit is as claimed.
Bonus Exercise. Show, using an ε-δ argument, that
lim x2 = 9.
x→3

3 Main Limit Theorem


One important tool for computing limits is the “Main Limit Theorem.” This is
essentially the same as the one we discussed for limits as x → ∞, but with the
following addition:
lim x = c.
x→c
In practice, when we’re computing limits as x → c, repeated applications of the
Main Limit Theorem usually end up just telling us that
lim f (x) = f (c).
x→c

When this holds for a function f , we say that f is continuous at c. (More on


this next lecture.)

7
Assignment 10 (due Wednesday, 26 October, 2011)
Consider the piecewise-linear function f defined by


3x + 11 if x ≤ −3,
f (x) = 1 − 31 x if − 3 < x ≤ 3,


3−x if x > 3.

1. Graph this function. Carefully.

2. Use the graph to “guess” what lim f (x) and lim f (x) are. If it looks like
x→−3 x→3
the two-sided limits don’t exist, sleep on it, and then try graphing the
function again.
3. Give ε-δ proofs that the two limits in the question above are what you
say they are. Remember: You should have one proof for each (two-sided)
limit. (Hint: you will probably want to let δ = min{δ1 , δ2 }, for appropriate
values of δ1 and δ2 .)
The third problem will be graded carefully.

Section 1.2, Problems 1–6. Problems 2, 4, and 6 will be graded carefully.

Section 1.3, Problems 1, 2, 5, and 6. Be sure to follow the instructions—these


problems are about using the Main Limit Theorem carefully, not about finding
the limits (those are very easy to guess). Problems 2 and 6 will be graded
carefully.

Assignment 11 (due Friday, 28 October, 2011)


• Section 1.5, Problems 51 and 52. Problem 52 will be graded carefully.
• Section 1.6, Problems 1, 3, and 5. None of these will be graded carefully.

• Do the exercise on page 2 of the notes for Lecture 12. This will be graded
carefully.
• Let a 6= 0 and c be arbitrary real numbers. (In other words, the “oppo-
nent” gets to choose a and c, and he is allowed to choose anything as long
as he does not set a equal to 0.) Give an ε-δ proof that

lim ax = ac.
x→c

This will be graded carefully.

8
• A “sequence” (an ) is a list of numbers, for instance,
1 3 7 15
0, , , , , . . . .
2 4 8 16
Typically, the nth term will be denoted an . Thus, in the sequence above,
we have

a1 = 0
1
a2 =
2
3
a3 =
4
7
a4 =
8
15
a5 =
16
..
.
2n−1 − 1
an =
2n−1
..
.

Explain why a “sequence” is the same thing as a “function with domain


the positive integers.” [Once you’ve thought about this for long enough,
it may become so obvious that you have very little to say. Unfortunately,
the grader cannot give you credit just for writing “It’s obvious.”]

• Bonus: Do the Bonus Exercise on page 7 of the Lecture 12 notes.

9
Math 131, Lecture 13
Charles Staats
Wednesday, 26 October 2011

1 An ε-δ proof for the limit of a piecewise-linear


function
Example. Let f be the function defined by
(
−7x + 8 if x ≤ 1,
f (x) =
2−x if x > 1.

f (x)

x
−4 −3 −2 −1 1 2 3 4
−1

−2

−3

−4

Give an ε-δ proof that


lim f (x) = 1.
x→1

Solution. First, since I don’t feel like doing an extensive preliminary analysis,
I’m going to let you in on a little trick: If you have a function that looks like

f (x) = mx + b,

1
with m 6= 0, and you want to do an ε-δ proof for f , then
1
δ= |m| ε

is a good guess for δ. I’m not saying it will always work, but it’s worth trying.
Now, for the function f in this particular example, we actually have two
linear equations. To the left of x = 1, we have f (x) = −7x + 8. This suggests
that for the left-hand limit, we should take
1
δ1 = |−7| ε = 17 ε.

To the right of x = 1, we have f (x) = −x + 1, which suggests that for the


right-hand limit, we should take
1
δ2 = |−1| ε = ε.

Since we’re interested in the two-sided limit, we should probably try


δ = min{δ1 , δ2 } = min{ 71 ε, ε} = 71 ε.
Now, let’s proceed to the actual proof, and see if this choice of δ works out.
Proof. Let ε > 0 be given. Set δ = 17 ε.
Assume 0 < |x − 1| < δ; we consider separately what happens when x < 1
and when x > 1.
Case 1: x < 1. Here, f (x) = −7x + 8, and so
|f (x) − `| = |f (x) − 1| = |−7x + 8 − 1|
= |−7x + 7|
= |7x − 7|
= 7|x − 1|
< 7δ
= 7 · 71 ε
= ε.

Case 2: x > 1. Here, f (x) = 2 − x, and so


|f (x) − 1| = |1 − x|
= |x − 1|

= 17 ε
< ε.

In either case, |f (x) − `| = |f (x) − 1| < ε, as desired.

2
2 Continuity
Given what we’ve already seen, the simplest definition of continuity is the fol-
lowing:
Definition. A function f is said to be continuous at a point x0 if
(i) f is defined at x0 , and

(ii) lim f (x) exists, and


x→x0

(iii) lim f (x) = f (x0 ).


x→x0

If f is continuous at every point of an interval, we say that f is continuous on


that interval.
If f is continuous at every point in its domain, we may say simply that f is
continuous.
Example. Consider the function f whose graph looks like this:

f (x)

x
−4 −3 −2 −1 1 2 3 4
−1

−2

−3

−4

1. At which points on the closed interval [−5, 5] is f not continuous?

f fails to be continuous at the x-values −3, 0, 1, and 3

2. At which points in its domain is f not continuous?

3
The x-values −3 and 1. The other x-values listed above do not lie in
the domain of f .

Example. The function g defined by g(x) = 1/x

g(x)

x
−4 −3 −2 −1 1 2 3 4
−1

−2

−3

−4

would be called continuous, since it is continuous on its domain; in other words,


it is continuous on the intervals (−∞, 0) and (0, ∞). [It is not continuous at
x = 0, but this “does not count” because 0 is not a point of its domain; f is not
defined at 0.]
The intuitive notion is that a function is continuous on an interval [a, b] if
you can draw f from the point (a, f (a)) to the point (b, f (b)) without picking
up your pencil. This intuitive idea is, unfortunately, something of a dead end if
we try to use this idea, rather than limits, to define what “continuous” ought
to mean. However, the following theorem does seem to capture the notion that
if you draw continuously from one point to another, you have to pass through
all the points in between:
Theorem. (Intermediate Value Theorem) Suppose f is continuous on the closed
interval [a, b]. Suppose we have a value y0 such that f (a) < y0 < f (b). Then f
hits the value y0 somewhere on the open interval (a, b). In other words, there
exists x0 such that a < x0 < b and f (x0 ) = y0 .

4
Here’s the picture:

f (b)
(b,f (b))

y0
(x0 ,y0 )
(a,f (a))
f (a)

a x0 b

Intuitively, for the (continuous) f to go from the line y = f (a) to the line
y = f (b), it has to pass through the line y = y0 somewhere. That “somewhere”
is our x0 .
We won’t try to prove this right now.

5
Assignment 11 (due Friday, 28 October, 2011)
• Section 1.5, Problems 51 and 52. Problem 52 will be graded carefully.
• Section 1.6, Problems 1, 3, and 5. None of these will be graded carefully.
• Do the exercise on page 2 of the notes for Lecture 12. This will be graded
carefully.

• Let a 6= 0 and c be real numbers (which you don’t get to choose). Give
an ε-δ proof that
lim ax = ac.
x→c

This will be graded carefully.


• A “sequence” (an ) is a list of numbers, for instance,
1 3 7 15
0, , , , , . . . .
2 4 8 16
Typically, the nth term will be denoted an . Thus, in the sequence above,
we have

a1 = 0
1
a2 =
2
3
a3 =
4
7
a4 =
8
15
a5 =
16
..
.
2n−1 − 1
an =
2n−1
..
.

Explain why a “sequence” is the same thing as a “function with domain


the positive integers.” [Once you’ve thought about this for long enough,
it may become so obvious that you have very little to say. Unfortunately,
the grader cannot give you credit just for writing “It’s obvious.”]
• Bonus: Do the Bonus Exercise on page 7 of the Lecture 12 notes. (This
exercise comes at the end of Section 2 of the Lecture 12 notes. If you want
to attempt it, you should probably read the entire section.)

6
Assignment 12 (due Monday, October 31, a.k.a.
Halloween)
Section 1.3, Problems 3, 4, 7, and 8. Remember, these problems are about
showing you understand how to use the Main Limit Theorem, not about finding
the limits. Problems 4 and 8 will be graded carefully.

Section 1.6, Problems 2, 4, 6-8, 32, and 33. Problems 6-8 and 32 will be graded
carefully. You do not need to give ε-δ proofs for these problems.

Bonus problem: Let f be a function (which you do not get to choose). Consider
the statement
For all x0 in the domain of f , for all ε > 0, there exists δ > 0 such
that whenever |x − x0 | < δ, then |f (x) − f (x0 )| < ε.
Explain why this is equivalent to the statement that “f is continuous.” (One
dilemma you may need to address: Why does it make no difference if we write
|x − x0 | < δ rather than 0 < |x − x0 | − δ?)

7
Math 131, Lecture 14
Charles Staats
Friday, 28 October 2011

1 Zeno’s arrow paradox


Wikipedia has a nice summary of Zeno’s arrow paradox:

In the arrow paradox (also known as the fletcher’s paradox), Zeno


states that for motion to occur, an object must change the position
which it occupies. He gives an example of an arrow in flight. He
states that in any one (durationless) instant of time, the arrow is
neither moving to where it is, nor to where it is not. It cannot
move to where it is not, because no time elapses for it to move
there; it cannot move to where it is, because it is already there. In
other words, at every instant of time there is no motion occurring.
If everything is motionless at every instant, and time is entirely
composed of instants, then motion is impossible.
This more or less captures the central conceptual idea in differential calculus.
When we have an object in motion, we’d like to be able to talk about how fast
it is going at any given instant. But the essence of motion is moving from one
position to another, whereas in a single, durationless instant, an object only
occupies a single position. So how can we even think about the speed at a
particular instant—or, to use slightly fancier terminology, the “instantaneous
velocity”?
There are basically two ways to think about this. The more mathematically
rigorous way is to use limits. The idea here is to say “since we can’t make the
change in time zero, let’s make it arbitrarily small.” Since we can’t touch the
(here) Zero beast, let’s handle it through the saddle of the Arbitrary.
The other way—the “walking on clouds” approach that was used for the
first two centuries or so after calculus was invented—is to say, in essence, “Let’s
pretend that the instant at time t0 actually does have an ‘infinitesimal’ duration,
which we call dt, and see what happens.” This “infinitesimal” duration, dt, is
bigger than zero, but smaller than any positive real number. The philosopher
Berkeley called such infinitesimals “ghosts of recently departed quantities.”
The textbook is of the opinion that the first, rigorous, approach is the only
way to go. Personally, I find the second approach extremely useful, even if it
is just “walking on clouds.” I also think you need to see it, since if you should

1
need calculus in applied science (physics, chemistry, atmospheric chemistry,. . . )
this is most likely the language you will see. But I’m honestly not sure which
approach is less confusing to see first, so I’m going to accept the following
wisdom: When in doubt, follow the textbook. More or less.

2 Defining instantaneous velocity


As said above, the essence of motion is changing from one position to another.
So, let’s suppose that an object changes its position over time. As Zeno pointed
out, at any given time, it has only one position. Thus, if we denote the object’s
position by x, then x is a function of the time t: there exists a function f such
that
x = f (t).
Consider what happens near a fixed time t0 . As a small amount of time elapses,
the object’s position changes by a small amount; the velocity is the change in
position divided by the change in time. For some reason, it is customary to
use the Greek letter ∆ (capital delta) to represent “change in.” Thus, with this
notation, the above sentence states that
∆x
velocity = .
∆t
There’s a bit of a problem here, though. If we specifically want the velocity at
the instant t0 , then we don’t have any change in time to work with: ∆t = 0.
Likewise, within the single instant, there is no change in position: ∆x = 0. So,
the expression above would tell us that velocity = 0/0. Since 0/0 is undefined,
this is not terribly helpful.
However, we have been studying a way to “fill in” such undefined values:
use limits. Thus, we define the instantaneous velocity at t0 , denoted dx/dt|t=t0 ,
to be
dx ∆x
= lim ,
dt t=t0 ∆x→0 ∆t
provided that this limit exists. The notation follows the convention that “when
you take a limit, you should replace Greek letters by Roman letters.” In this
case, we replace the Greek letter ∆ by the Roman letter d.
Recall that the object starts at time t0 . If the time changes by ∆t = h, then
the corresponding change in position is ∆x = f (t0 + h) − f (t0 ). Thus, the above
equation can also be written
dx f (t0 + h) − f (t0 )
= lim .
dt t=t0 h→0 h
Finally, if we bring to bear all of the different notations we’re likely to use for
this, we’ll get
df dx f (t0 + h) − f (t0 )
f 0 (t0 ) = (t0 ) = = lim .
dt dt t=t0 h→0 h

2
This quantity (when it exists) is called the derivative of f at t0 .

3 A few more bits on continuity


People generally learn stuff better if they see it over a period of time, rather
than all at once. Thus, I’ve decided to distribute the subject of continuity over
multiple lectures.1
Theorem. Every polynomial or rational function is continuous on its natural
domain. The same holds if you throw in nth roots.
The proof of this theorem is by repeated applications of the Main Limit
Theorem. I won’t try to give the complete proof, but I will give you an example.
Example. Show, using the Main Limit Theorem, that the function f defined
by √
1 + 2x
f (x) = 3
x − 13
is continuous.
Solution. For every real number c such that f (c) is defined, we have

1 + 2x
lim f (x) = lim 3
x→c x→c x − 13

limx→c 1 + 2x
=
limx→c x3 − 13

1 + limx→c 2x
=
c3 − 13

1 + 2c
= 3 = f (c).
c − 13

By hypothesis, f (c) is defined, i.e., the last line makes sense. By the Main Limit
Theorem, the previous line makes sense and is equal to it, and so on all the way
up. Thus, for every c in the domain of f ,

lim f (x) = f (c).


x→c

In other words, f is continuous.


Finally, an example of using the Intermediate Value Theorem that hearkens
back to the bonus problem from the first lecture:

Example. Show that 3 31 exists. In other words, show that there is a positive
real number x0 such that x30 = 31.
1 Translation: I did not really get to say all I wanted on continuity last lecture, so I’m

trying to cram in a few extra notes at the end of this one.

3
Solution. a

64
(4, 64)

31

0
x0 4
0

Let f be the function defined by f (x) = x3 . Since f is a polynomial function,


f is continuous on its domain (−∞, ∞), and in particular on the interval [0, 4].
Observe that
f (0) = 0 < 31 < 64 = f (4).
Hence, there exists some x0 such that 0 < x0 < 4 and f (x0 ) = 31.

4
Assignment 12 (due Monday, October 31, a.k.a.
Halloween)
Section 1.3, Problems 3, 4, 7, and 8. Remember, these problems are about
showing you understand how to use the Main Limit Theorem, not about finding
the limits. Problems 4 and 8 will be graded carefully.

Section 1.6, Problems 2, 4, 6-8, 32, and 33. Problems 6-8 and 32 will be graded
carefully. You do not need to give ε-δ proofs for these problems.

Bonus problem: Let f be a function (which you do not get to choose). Consider
the statement
For all x0 in the domain of f , for all ε > 0, there exists δ > 0 such
that whenever |x − x0 | < δ, then |f (x) − f (x0 )| < ε.
Explain why this is equivalent to the statement that “f is continuous.” (One
dilemma you may need to address: Why does it make no difference if we write
|x − x0 | < δ rather than 0 < |x − x0 | − δ?)

Assignment 13 (due Wednesday, 2 November)


NOTE: From this assignment on, you no longer need to write anything about
“By the Main Limit Theorem,. . . ” when showing your work to take a limit.
(You should, however, continue to show your work.)

2
Use the Intermediate Value Theorem √ to prove that, no matter what Diophantus
of Alexandria might have thought, 2 does, in fact, exist. (In other words, there
exists a positive real number x0 such that x20 = 2.) This problem will be graded
carefully.

Section 2.2, Problems 45-48 and 51, 52. Be sure to follow the instructions
carefully on 51 and 52; these problems are as much about how you find the
derivative, as what answer you get. Problems 46, 48, and 52 will be graded
carefully.

Let a 6= 0 be a real number (which you don’t get to choose). Let f be the
function defined by
f (x) = ax.
Show that f is continuous using an ε-δ proof. This problem will be graded
carefully.

2 See Lecture 3, page 2.

5
Math 131, Lecture 15
Charles Staats
Monday, 31 October 2011

1 The derivative as the slope of the tangent line


In classical geometry, the tangent to a curve was the line that somehow “touched
the curve without crossing it.” Euclid attempted to make this precise by de-
scribing the tangent as the line that intersected the curve in only one point. His
definition works quite well for circles (and also ellipses, parabolas, and hyper-
bolas):

However, it can fail rather drastically for more complicated curves. In the
curve below, the almost-vertical line is the one that intersects the curve in only
one point, while the almost-horizontal line clearly “ought” to be the tangent
line. (Intuitively, the almost-vertical line crosses the curve, while the almost-
horizontal line does not—at least, not at the point in question.)

1
For another example, in the following picture, neither the vertical nor the hori-
zontal line really “touches the curve without crossing it.” Each of them intersects
the curve exactly once. But if one of them is the tangent line, it is the horizontal
line rather than the vertical line.

Thus, we take another approach to defining what exactly the tangent line
should be. An easier definition is to define a secant line—that is, a line that
passes through two specified points on a curve. This is easy to specify, since
two points determine a line. We want to think of a tangent line as a “secant
line that passes through the same point twice.” Unfortunately, this does not
actually make any sense.
To remedy the situation, we consider another way of specifying a line: a
point (x0 , y0 ) together with a slope ∆y/∆x. Thus, the secant line through
(x0 , y0 ) and (x, y) is the line passing through (x0 , y0 ) with slope equal to

∆y y − y0
= .
∆x x − x0
If we want to take the tangent line at (x0 , y0 ), we already have a point through
which the line should pass. We just need to know what its slope ought to be.
This is essentially the same problem we were faced with last lecture—we need
a definition for “slope at a point,” in spite of the fact that slope is, inherently, a
property relating two different points. And we solve it the same way: we take
a limit. We say that the slope of the tangent line is
∆y
lim .
∆x→0 ∆x

2
The picture below shows the tangent line as a limit of secant lines:

(x0 ,y0 )

Now, if you recall the previous definition of the derivative, you will see that, if
y is given as y = f (x) for some function f , then in fact, we will have the slope
of the tangent line equal to the derivative:

∆y dy
lim = = f 0 (x0 ).
∆x→0 ∆x dx x=x0

This gives us the following

Definition. Let f be a function defined at x0 . The tangent line to f at x0 is


the line passing through the point (x0 , f (x0 )) and having slope equal to f 0 (x0 ),
provided that this derivative exists.
Let’s do an example.
Example. Let the function f be defined by

f (x) = x2 .

Compute the derivative of f at x0 = 1. Plot the function and the line tangent
to f at x0 .
Solution. First, let’s solve for ∆y in terms of ∆x:

∆y = f (x0 + ∆x) − f (x0 )


= (1 + ∆x)2 − 12
= 1 + 2∆x + (∆x)2 − 1
= 2∆x + (∆x)2 .

3
Thus, we have
∆y
f 0 (x0 ) = lim
∆x
∆x→0
2∆x + (∆x)2
= lim
∆x→0 ∆x
= lim 2 + ∆x
∆x→0
= 2.

Now, we plot the function y = f (x), together with line passing through
(x0 , f (x0 )) = (1, 1) and having slope f 0 (x0 ) = 2:
y

(x0 ,f (x0 ))

2 Infinitesimals
The idea of infinitesimals, as it relates to slopes of tangent lines, is to define
the tangent line to f at x0 as the line through (x0 , y0 ) and another point (x0 +
dx, y0 + dy) that is “infinitely close” to the first point. What this means, in
this example, is that dx is “so small” that dx2 = 0, even though dx is not zero.

4
This sort of makes sense, in that the square of a small number is a much smaller
number; for instance,
0.0012 = 0.000001
It does not really make sense—no nonzero number can square to zero—but
that’s why I called this “walking on clouds.”
To start with, we treat the “infinitesimal changes” dx and dy exactly as
though they were more conventional changes ∆x and ∆y. Our earlier compu-
tation of ∆y in terms of ∆x still holds:

dy = 2dx + dx2
dy = 2dx since dx2 = 0
dy
=2
dx
when evaluated at the point x0 = 1.

5
Assignment 13 (due Wednesday, 2 November)
NOTE: From this assignment on, you no longer need to write anything about
“By the Main Limit Theorem,. . . ” when showing your work to take a limit.
(You should, however, continue to show your work.)

1
Use the Intermediate Value Theorem √ to prove that, no matter what Diophantus
of Alexandria might have thought, 2 does, in fact, exist. (In other words, there
exists a positive real number x0 such that x20 = 2.) This problem will be graded
carefully.

Section 2.2, Problems 45-48 and 51, 52. Be sure to follow the instructions
carefully on 51 and 52; these problems are as much about how you find the
derivative, as what answer you get. Problems 46, 48, and 52 will be graded
carefully.

Let a 6= 0 be a real number (which you don’t get to choose). Let f be the
function defined by
f (x) = ax.
Show that f is continuous using an ε-δ proof. This problem will be graded
carefully.

Assignment 14 (due Friday, 4 November)


Section 2.2, Problems 37–44. The even-numbered problems will be graded care-
fully.

For each of Problems 1–4 in Section 2.2, do the following steps:

(a) Find the indicated derivative using infinitesimals.


(b) Find the indicated derivative using the limit definition.
(c) Graph the function together with the tangent line at the indicated point.

Problems 2 and 4 will be graded carefully.

1 See Lecture 3, page 2.

6
Math 131, Lecture 16
Charles Staats
Wednesday, 1 November 2011

1 Some alternate ways to state the limit defini-


tion of the derivative
For this section, we’re going to use the notation f 0 (x0 ) rather than dy/dx|x=x0 .
Our basic definition of the derivative has been

∆y
f 0 (x0 ) = lim . (1)
∆x→0 ∆x

One key to mastering mathematics is being able to move facilely among dif-
ferent ways of saying the same thing; which way you want to say it may
depend on what you want to use it for. We’re going to review some other
ways to write the definition of the derivative, using the various relations among
x, x0 , ∆x, y, ∆y, f (x0 ), . . ..
First, observe that

∆y = y − y0 = f (x) − f (x0 ) and


∆x = x − x0 .

Thus,
∆y f (x) − f (x0 )
= ,
∆x x − x0
and
∆y
lim =`
∆x→0 ∆x
∆y
⇐⇒ ∀ε > 0, ∃δ > 0 s.t. if 0 < |∆x − 0| < δ, then −` <ε
∆x
f (x) − f (x0 )
⇐⇒ ∀ε > 0, ∃δ > 0 s.t. if 0 < |x − x0 | < δ, then −` <ε
x − x0
f (x) − f (x0 )
⇐⇒ lim = `.
x→x0 x − x0

1
In other words, an alternate definition for the derivative is given by

f (x) − f (x0 )
f 0 (x0 ) = lim . (2)
x→x0 x − x0

This definition highlights the feature that the derivative only depends on what
is happening to f near x0 . If we look at a different function g that cannot be
distinguished from f near x0 , then f and g will have the same derivative at x0 ;
i.e., f 0 (x0 ) = g 0 (x0 ).
Another way to state the definition of the derivative is to express ∆y in
terms of x0 and ∆x, rather than x0 and x.

∆y = f (x) − f (x0 )
= f (x0 + ∆x) − f (x0 ),

since x = x0 + ∆x. Thus, we have


∆y f (x0 + ∆x) − f (x0 )
f 0 (x0 ) = lim = lim .
∆x→0 ∆x ∆x→0 ∆x
Making the traditional change of notation ∆x = h, we find that

f (x0 + h) − f (x0 )
f 0 (x0 ) = lim . (3)
h→0 h

The expression inside the limit is the infamous “difference quotient.”

2 Derivative as a function
In the definition of (3), one feature is that there are no appearances of the letter
x except in the variable x0 . Thus, we can rename x0 as x, obtaining
f (x + h) − f (x)
f 0 (x) = lim .
h→0 h
The interesting feature here is that when we rewrite the definition this way,
it becomes obvious that we have defined more than a number f 0 (x0 ); we have
defined a function f 0 .
There’s a subtlety here that confused me when I first saw this sort of thing.
It involves the interplay of intuition and rigorous mathematics. Intuitively,
when we write x, we think of it as a variable—something that is allowed to
range over many different numbers. On the other hand, when we write x0 ,
we think of this as a particular value of x, a particular number; we just don’t
happen to know what number it is. These intuitions are valuable. However, it
is equally valuable to realize that these intuitions have absolutely no reflection
in the rigorous mathematics. As far as the pure logic is concerned, x and x0 are
both variables, and that’s all there is to it. So whenever we have a statement

2
that involves only one, we can substitute the other, and get an equally true
expression that feels very different, intuitively.
This is typical of a certain kind of reasoning that appears sometimes in
mathematics. First, you let your intuition guide you, as we did (more or less) in
defining the derivative. Then you do something with rigorous mathematics to
change the statement into something equivalent, but that feels intuitively very
different. At this point, you may feel like your head wants to explode: your
intuition is screaming that what you’ve done can’t possibly be right, but you
can’t see any flaws in your logic. It may be tempting to give up and think about
something else. But instead, you may force yourself to stay on task, to turn
the thing over and over in your head until you either find a flaw in the logic, or
find a way of thinking about it that your intuition will accept. Depending on
the difficulty of the thing in question, resolving the conflict may take moments,
hours, days, weeks, months, or years. But the longer you spend puzzling over
it, the greater will be your feeling of enlightenment when it finally “clicks.”
On the other hand, some of you may be thinking that it was obvious that
the derivative is a function. You may even feel a bit smug about the fact that
this “revelation” was clear to you from the beginning. Perhaps you should. But
I think it is more likely that you were not following my lectures closely, but
were instead thinking about the derivative in terms you have learned in the
past. Or perhaps you never really understood the intuition of x0 as a “fixed
value we don’t know,” versus x as a “variable.” Either way, I suggest you review
the previous buildup to the definition of the derivative. Try to understand with
your whole mind—both logic and intuition. If you succeed, you may get a part
of the revelatory moment that you will otherwise have been cheated of.
Now, enough philosophizing. Since we’ve established that the derivative f 0
is a function, there are two obvious sorts of questions:
1. How do we find a formula for the function, if one exists?
2. How do we characterize the function, even if it does not have a formula
we can write down?

We’ll spend a lot of time on both of these, but in light of the homework I’ve
assigned you for Friday, I’m going to spend the rest of this lecture on a version of
the second problem. Specifically: If someone gives you a graph of the function,
how do you graph its derivative? We’ll approach this mainly through examples.
My plan (which I may or may not have time for) is to give you a few minutes to
try the following examples on your own, and then we will go over them together.

3
Example. The graph of a function f is given on the left. On the right, sketch
the graph of the function f 0 . Remember: above each point x on the x-axis, the
value of f 0 should be the slope of the tangent line to f at x. If f does not have
a unique tangent line at x, then f 0 (x) will not exist.
f (x) f 0 (x)

4 4

3 3

2 2

1 1

x x
−4 −3 −2 −1 1 2 3 4 −4 −3 −2 −1 1 2 3 4
−1 −1

−2 −2

−3 −3

−4 −4

f (x) f 0 (x)

4 4

3 3

2 2

1 1

x x
−4 −3 −2 −1 1 2 3 4 −4 −3 −2 −1 1 2 3 4
−1 −1

−2 −2

−3 −3

−4 −4

f (x) f 0 (x)

4 4

3 3

2 2

1 1

x x
−4 −3 −2 −1 1 2 3 4 −4 −3 −2 −1 1 2 3 4
−1 −1

−2 −2

−3 −3

−4 −4

4
3 Local nature of the limit (and derivative)
I’m probably not going to have time to really go over this section in the lecture,
but I would feel like I would not be fulfilling my responsibilities as a Math 131
teacher if I did not at least mention it in the lecture notes.
Recall that, in the most vague terms, the statement

lim f (x) = `
x→x0

means something like “when x is near x0 , then f (x) is near `.” Thus, it seems
like this limit should only depend on “what f is doing near x0 .” In particular,
it should only depend on how f behaves on an interval (x0 − ∆x, x0 + ∆x).
y
g

`
f
x
x0 − ∆x x0 x0 + ∆x g

f = g on this interval, so
lim f (x) = lim g(x)
x→x0 x→x0

The way we say that the limit “only depends on what f is doing near x0 ” is
that if we replace f by a different function g that “looks the same near x0 ,”
then we are guaranteed to get the same answer. More precisely, we have the
following theorem:
Theorem. Suppose that f and g are two functions. Let ∆x be positive. If f
and g are defined and agree on the interval (x0 − ∆x, x0 + ∆x), then

lim f (x) exists if and only if lim g(x) exists.


x→x0 x→x0

Moreover, if the two limits exist, then they are equal.


Proof. Assume that
lim f (x) = `.
x→x0

We will then show that limx→x0 g(x) = `.


Let ε > 0 be given.

5
Since limx→x0 f (x) = `, there exists δ1 > 0 such that if 0 < |x − x0 | < δ1 ,
then |f (x) − `| < ε. Set δ = min{δ1 , ∆x}.
Assume 0 < |x − x0 | < δ. Since |x − x0 | < δ ≤ ∆x, we know f (x) = g(x).
Consequently,

|g(x) − `| = |f (x) − `|
< ε,

since 0 < |x − x0 | < δ ≤ δ1 .


Therefore,
lim g(x) = `,
x→x0

as claimed.
Similar reasoning shows that, if

lim g(x) = `,
x→x0

then limx→x0 f (x) = `.


Since one definition for the derivative is
dy f (x) − f (x0 )
= lim ,
dx x=x0
x→x0 x − x0

the theorem tells us that the derivative of f at x0 depends only on how f behaves
near x0 .

6
Assignment 14 (due Friday, 4 November)
Section 2.2, Problems 37–44. The even-numbered problems will be graded care-
fully.

For each of Problems 1–4 in Section 2.2, do the following steps:


(a) Find the indicated derivative using infinitesimals.

(b) Find the indicated derivative using the limit definition.


(c) Graph the function together with the tangent line at the indicated point.
Problems 2 and 4 will be graded carefully.

Assignment 15 (due Monday, 7 November)


Section 2.2, Problems 5–8 and 51–54. Follow the instructions. Remember, these
problems are more about how you find the derivative than what derivative you
find. The even-numbered problems will be graded carefully.

Section 2.3, Problems 1-4. These will not be graded carefully.

Draw a picture that explains why the difference quotient

f (x + h) − f (x)
h
gives the slope of a secant line to the curve y = f (x). Hint: your intuition may
like this problem better if you think in terms of x0 rather than x. This problem
will be graded carefully.

7
Math 131, Lecture 17
Charles Staats
Friday, 4 November 2011

1 Some example computations


We’re going to do compute some derivatives as functions using the definition

f (x + h) − f (x)
f 0 (x) = lim .
h→0 h
Example. Suppose that f (x) = x. Compute a formula for the function f 0 .

Solution.
f (x + h) − f (x)
f 0 (x) = lim
h→0 h
(x + h) − x
= lim
h→0 h
h
= lim
h→0 h
= lim 1
h→0
= 1.

Example. Suppose that f (x) = mx + b. Since y = f (x) is a line, the tangent


line will be the line itself; its slope, of course, is m. Thus, we may suppose that
f 0 (x) = m for all x. Prove this using the limit definition.

1
Solution.
f (x + h) − f (x)
f 0 (x) = lim
h→0 h
m(x + h) + b − (mx + b)
= lim
h→0 h
mx + mh + b − mx − b
= lim
h→0 h
mh
= lim
h→0 h
= lim m
h→0
= m.

Example. Let f be the function defined by f (x) = x2 + x − 3. Compute a


formula for the function f 0 .

Solution.
f (x + h) − f (x)
f 0 (x) = lim
h→0 h
(x + h)2 + (x + h) − 3 − x2 − x + 3
= lim
h→0 h
x2 + 2xh + h2 + x + h − 3 − x2 − x + 3
= lim
h→0 h
2
2xh + h + h
= lim
h→0 h
(2x + h + 1)h
= lim
h→0 h
= lim 2x + h + 1
h→0
= 2x + 1.

Example. Let f be the function defined by f (x) = 1/x. Compute a formula


for the derivative of f (except at x = 0, of course).

2
Solution.
f (x + h) − f (x)
f 0 (x) = lim
h→0 h
1 1
− x(x + h)
= lim x+h x ·
h→0 h x(x + h)
x − (x + h)
= lim
h→0 hx(x + h)
−h
= lim
h→0 hx(x + h)
−1
= lim
h→0 x(x + h)
−1
= 2.
x

Notation. It can be rather tiresome to write, for instance, “the derivative of


the function f defined by f (x) = x2 + x − 3.” In the future, we will sometimes
abbreviate this by
d 2
(x + x − 3).
dx

2 Product rule
Suppose that we have u and v, two functions of x. Suppose we know how to
calculate the derivatives du/dx and dv/dx. We can use this to calculate the
derivative of the product u · v, by means of the product rule.
Warning. It may be tempting to write that
d du dv
(u · v) = · .
dx dx dx
This is not true. For instance, suppose u(x) = 2 and v(x) = x. Then

d d
(u · v) = (2 · x) = 2,
dx dx
since y = 2x is a line of slope 2. However, the “naive product rule” would give
us
d d d
(2 · x) = (2) · (x) = 0 · 1 = 0.
dx dx dx
The naive product rule gives the wrong answer.
Leibniz gave a cute derivation of the product rule using infinitesimals. The
first equation in this proof may seem a bit confusing at first; I’ll explain it

3
du v du

u dv
u uv

v dv

Figure 1: A visual illustration of the product rule. The area of the white
rectangle is uv; the area of the total rectangle is (u + du)(v + dv); and the
change in area, d(uv), is their difference. The black rectangle, with area du dv,
is so small that its contribution is “can be neglected.”

afterwards, but if I give it now, the proof will not seem so “cute.” Remember,
the key “fact” about infinitesimals is that if you multiply two of them together,
you get something “doubly infinitesimal,” which we typically consider equal to
zero. In particular, du dv = 0.

d(uv) = (u + du)(v + dv) − uv


= uv + u dv + v du + du dv − uv
= u dv + v du.

Dividing through by dx, we see that


d dv du
(uv) = u +v .
dx dx dx
Now, the promised explanation of the first line: we have two functions u
and v of x. But we really have three functions: the one we care about is the
function f defined by f = uv, i.e.,

f (x) = u(x) · v(x).

Thus,

df = f (x + dx) − f (x)
= u(x + dx)v(x + dx) − u(x)v(x).

4
Recall that

du = u(x + dx) − u(x), hence


u + du = u(x + dx).

Similarly, v + dv = v(x + dx), and so we have

df = u(x + dx)v(x + dx) − u(x)v(x)


= (u + du)(v + dv) − uv.

(By an abuse of notation, we’re writing things like u for u(x) when it suits us
to do so.)
Example. Use the product rule to find (in this order) the derivatives of x2 , x3 ,
and x4 with respect to x.

Solution.
d 2 d
x = (x · x)
dx dx
dx dx
=x +x
dx dx
=x+x
= 2x.

d 3 d
x = (x · x2 )
dx dx
d d
= x (x2 ) + x2 (x)
dx dx
d 2
We just calculated dx (x ) = 2x, so this is equal to

= x · 2x + x2 · 1
= 2x2 + x2
= 3x2 .

d 4 d
x = (x · x3 )
dx dx
d 3 d
=x· (x ) + x3 (x)
dx dx
= x · 3x2 + x3 · 1
= 3x3 + x3
= 4x3 .

5
You may start to notice a pattern here. This pattern will continue: if we
d n−1
calculate on out to dx x , we’ll find that it is equal to (n − 1)xn−2 . Using this
fact, we find that
d n d
x = (x · xn−1 )
dx dx
d n−1 d
=x· (x ) + xn−1 (x)
dx dx
= x · (n − 1)xn−2 + xn−1 · 1
= (n − 1)xn−1 + xn−1
= nxn−1 ,

so the pattern always keeps going. (This is a version of “proof by induction.”)

6
Assignment 15 (due Monday, 7 November)
Section 2.2, Problems 5–8 and 51–54. Follow the instructions. Remember, these
problems are more about how you find the derivative than what derivative you
find. The even-numbered problems will be graded carefully.

Section 2.3, Problems 1-4. These will not be graded carefully.

Draw a picture that explains why the difference quotient


f (x + h) − f (x)
h
gives the slope of a secant line to the curve y = f (x). Hint: your intuition may
like this problem better if you think in terms of x0 rather than x. This problem
will be graded carefully.

Assignment 16 (due Wednesday, 9 November)


Section 2.2, Problems 11–14 and 55–58. Follow the instructions. Remember,
these problems (except for 57 and 58) are more about how you find the derivative
than what derivative you find. The even-numbered problems will be graded
carefully (although a certain amount of leeway will be provided on problem 58).

Section 2.3, Problems 11–14 and 23–26. (Hint: 23–26 are easier if you use the
product rule.) The even-numbered problems will be graded carefully.

Translate the statement


∆y
lim = f 0 (x0 )
∆x→0∆x
into ε-δ language. (Hint: when you see f 0 (x0 ), treat it like `. Also, treat ∆y/∆x
as a function of ∆x.) Then, use the resulting statement to prove the following:

∀ε > 0, ∃δ > 0 s.t. if |∆x| < δ, then ∆y is within ε of f 0 (x0 )∆x.

You will need to handle ∆x = 0 as a separate case. This statement is a rigorous


version of the statement that “When ∆x is small, then ∆y is approximately
dy
dx ∆x.”

Test II around Wednesday, 16 November


Experienced teachers of Math 131 tell me that you will probably have a lot of
papers and the like due around the time of the test, and consequently will not
have a lot of time to study for it. Thus, I suggest you start studying now. You
may also want to think in terms of “practicing” rather than “studying”: redoing
old quiz and homework problems (without looking at the solutions, if you have
them, until afterwards) may be more helpful than simply reading over them.

7
Math 131, Lecture 18: Rules for differentiation
Charles Staats
Monday, 7 November 2011

The process of taking a derivative, often called “differentiation,” is extremely


important. Moreover, unlike many important things in mathematics, differen-
tiation is actually possible to do. Any time you have a function given by a
formula, the rules in this lecture will allow you to find its derivative.
These rules need to be memorized. Ideally, they should become so ingrained
that you can use them without having to think about them.

1 The “easy rules”


There are a few “easy” rules for differentiation.

Theorem. (Constant rule) If f is the function defined by f (x) = c, where c is


a (constant) real number, then f 0 (x) = 0 for all x.
Proof. This is a special case of the mx + b rule we proved last time, but let’s do
it again anyway.

f (x + h) − f (x)
f 0 (x) = lim
h→0 h
c−c
= lim
h→0 h
0
= lim
h→0 h
= 0.

Theorem. (Sum rule) If f and g are differentiable functions, then

(f + g)0 = f 0 + g 0 .

In words, “the derivative of a sum is the sum of the derivatives.”

1
Proof. We apply one of the limit definitions of the derivative:

(f + g)(x + h) − (f + g)(x)
(f + g)0 (x) = lim
h→0 h
f (x + h) + g(x + h) − f (x) − g(x)
= lim
h→0 h
f (x + h) − f (x) g(x + h) − g(x)
= lim +
h→0 h h
= f 0 (x) + g 0 (x)
= (f 0 + g 0 )(x).

Since this holds for all x at which f and g are defined, we have the equality of
functions
(f + g)0 = f 0 + g 0 .
Theorem. (Multiplication by a constant) If f is a differentiable function of x
and c is a (constant) real number, then

d df
(cf (x)) = c .
dx dx
Proof. This is, again, a special case of the mx + b thing. This time, we’re going
to derive it from the product rule.
d df d
(cf (x)) = c + f (x) (c)
dx dx dx
df
=c + f (x) · 0 (constant rule)
dx
df
=c .
dx
Theorem. (Difference rule) If f and g are differentiable functions, then (f −
g)0 = f 0 − g 0 .

Proof.
0
(f − g)0 = f + (−1) · g
0
= f 0 + (−1)g (sum rule)
0 0
= f + (−1)g (multiplication by a constant)
= f 0 − g0 .

2 The power rule; polynomials


The power rule is fairly easy, but a bit less intuitive than the “easy rules.” It
was mentioned briefly at the end of the last lecture.

2
Theorem. When n is a positive integer,
d n
x = nxn−1 .
dx
(Actually, this theorem applies whenever n is a real number, but we won’t
be able to prove that for some time.)

To understand the proof of the Power Rule, we need a technique called


mathematical induction. Suppose we have a condition P (n) on n. The
“induction principle” says that to show P (n) is true whenever n is a positive
integer, we can do show the following:
1. P (1) is true.
2. Whenever P (n) is true, then P (n + 1) is also true.
Thus, P (1) is true; since P (1) is true, P (2) is also true; since P (2) is true,
P (3) is also true; and so on.
One standard metaphor here is that in step 2, we set up a chain of
dominoes; in step 1, we knock over the first one, which then knocks over
the second one, which then knocks over the third one, etc.

Proof. Let P (n) be the statement that Dx (xn ) = nxn−1 ; this is a condition
on n.
1. We first show that P (1) is true, i.e., that Dx (x) = 1:

d (x + h) − h
(x) = lim
dx h→0 h
h
= lim
h→0 h
= 1,

as desired.
2. We now show, using the product rule, that whenever P (n) is true, then
P (n + 1) is also true.

d n+1 d
x = (x · xn )
dx dx
d d
= x (xn ) + xn (x) (product rule)
dx dx
= x · nxn−1 + xn · 1 (since P (n) is true)
= nxn + xn
= (n + 1)xn .

3
Thus, by induction, P (n) is true for every positive integer n. In other words,
for every positive integer n,
d n
x = nxn−1 .
dx
Using the power rule, together with the “easy rules,” we can, in principle,
compute the derivative of any polynomial.
Example 1. Differentiate x2 − 4x + 1.

Solution.
d 2 d 2 d d
(x − 4x + 1) = (x ) − (4x) + (1) (sum rule)
dx dx dx dx
d 2 d
= (x ) − 4 (x) + 0 (constant multiple; constant)
dx dx
= 2x − 4 · 1 + 0 (power rule)
= 2x − 4.

Example 2. Differentiate 2x3 − 21 x2 − x + 17246


937 .

Solution.
d 
2x3 − 12 x2 − x + 17246
937 = 2 · 3x2 − 1
2 · 2x − 1 + 0
dx
= 6x2 − x − 1.

3 Proof of the product rule


Recall the product rule,
d dv du
(uv) = u +v ,
dx dx dx
and the (non-rigorous) infinitesimal derivation:

d(uv) = (u + du)(v + dv) − uv


= uv + u dv + v du + du dv − uv
= u dv + v du
d dv du
(uv) = u +v .
dx dx dx

4
We are now going to show how to prove the product rule rigorously. Pay at-
tention to how what we are doing rigorously corresponds to the non-rigorous
infinitesimal method.
Theorem. Suppose that u is a function of x such that du/dx|x=x0 exists. Like-
wise, suppose that v is a function of x such that dv/dx|x=x0 exists. Then the
derivative of the product uv at x0 exists, and
d dv du
(uv) = u0 + v0 .
dx x=x0 dx x=x0 dx x=x0

Proof. First, we write ∆(uv) in terms of ∆u and ∆v:


∆(uv) = uv − u0 v0
= (u0 + ∆u)(v0 + ∆v) − u0 v0
= u0 vu + u0 ∆v + v0 ∆u + ∆u∆v − u0 v0
= u0 ∆v + v0 ∆u + ∆u∆v.
[Notice how closely this resembles the infinitesimal version.] Now, we apply the
definition1 of the derivative as a limit:
d ∆(uv)
(uv) = lim
dx x=x0
∆x→0 ∆x
u0 ∆v + v0 ∆u + ∆u∆v
= lim
∆x→0 ∆x
∆v ∆u ∆u ∆v
= lim u0 + v0 + · · ∆x
∆x→0 ∆x ∆x ∆x ∆x  
dv du du dv
= u0 + v0 + x=x0 x=x0 ·0
dx x=x0 dx x=x0 dx dx
dv du
= u0 + v0 .
dx x=x0 dx x=x0

Note the trick on the third line that was used to show that ∆u∆v/∆x → 0:
∆u∆v ∆u∆v∆x ∆u ∆v du dv
= = · · ∆x → · ·0=0
∆x (∆x)2 ∆x ∆x dx dx
as ∆x → 0. This (sort of) gives a justification for the infinitesimal idea that
du dv = 0.

4 The Chain Rule


Arguably the most important of all of these rules is the chain rule, which tells
us how to take derivatives of compositions of functions. It states that if f and
g are differentiable functions, then
(f ◦ g)0 (x) = f 0 (g(x)) · g 0 (x).
1 Or rather, one of the equivalent definitions.

5
We can give a non-rigorous, infinitesimal derivation as follows: One (non-
rigorous) definition of the derivative is that, if y = f (x), then f 0 (x) is the
number such that
dy = f 0 (x)dx.
Now, suppose that y = f (u) and u = g(x), so that y = f (u) = f (g(x)). Then
we have dy = f 0 (u)du and du = g 0 (x)dx, so

dy = f 0 (u)du
= f 0 (u)g 0 (x)dx
= f 0 (g(x))g 0 (x)dx.

Hence,
dy
= f 0 (g(x))g 0 (x).
dx
Example 3. Differentiate (x + 1)500 .
Solution.
d d
(x + 1)500 = 500(x + 1)499 · (x + 1)
dx dx
= 500(x + 1)499 · 1
= 500(x + 1)499 .

It would have been possible, but very hard, to differentiate this by expanding
out all 501 terms of the polynomial and then applying the techniques of the first
section.

6
Assignment 16 (due Wednesday, 9 November)
Section 2.2, Problems 11–14 and 55–58. Follow the instructions. Remember,
these problems (except for 57 and 58) are more about how you find the derivative
than what derivative you find. The even-numbered problems will be graded
carefully (although a certain amount of leeway will be provided on problem 58).

Section 2.3, Problems 11–14 and 23–26. (Hint: 23–26 are easier if you use the
product rule.) The even-numbered problems will be graded carefully.

Translate the statement


∆y
lim = f 0 (x0 )
∆x→0∆x
into ε-δ language. (Hint: when you see f 0 (x0 ), treat it like `. Also, treat ∆y/∆x
as a function of ∆x.) Then, use the resulting statement to prove the following:

∀ε > 0, ∃δ > 0 s.t. if |∆x| < δ, then ∆y is within ε|∆x| of f 0 (x0 )∆x.

You will need to handle ∆x = 0 as a separate case. This statement is a rigorous


version of the statement that “When ∆x is small, then ∆y is approximately
dy
dx ∆x.”

Assignment 17 (due Friday, 11 November)


From Section 2.3:
• Problems 5–8. Do each problem two ways—using the limit definition of
your choice, and using the rules of differentiation (including the Chain
Rule, if you find it helpful).
• Problems 17–20.
• Problems 31–32. Do not FOIL out the products; instead, use the product
rule for differentiation.
The even-numbered problems will be graded carefully.

Section 2.5, Problems 1–4. Make sure it is clear, from your answer, how you are
using the Chain Rule (see, for instance, Example 3 at the end of Lecture 18).
Problems 2 and 4 will be graded carefully.

Give an ε-δ proof for each of the following:


1. Let f be the function defined by
(
7x − 3 if x ≤ 0,
f (x) =
− 19 x − 3 if x > 0.

Show that lim f (x) = −3.


x→0

7
2. Let f be the function defined by
(
−1x − 18
if x < 3,
f (x) = 1 7 7 7

6x − 2 if x ≥ 3.

Show that lim f (x) = −3.


x→3

3. Let f be the function defined by



 1 3
− 2 x + 2 if x < −3,
f (x) = 4 if x = −3,


3x + 12 if x > −3.

Show that lim f (x) = 3.


x→−3

Problems 1 and 3 will be graded carefully.

Suppose y = f (x) and f (x0 ) = y0 . A purely ε-δ version of the statement that
“f is continuous at x0 ” is given as follows:

∀ε > 0, ∃δ > 0 s.t. if |x − x0 | < δ, then |y − y0 | < ε.

Use this definition to prove the following fact:


Suppose that

u = f (x),
u0 = f (x0 ),
y = g(u) = g(f (x)), and
y0 = g(u0 ) = g(f (x0 )).

If f is continuous at x0 and g is continuous at u0 , then g ◦ f is


continuous at x0 .
This will be graded carefully.

Part of Assignment 18
Assignment 18 will include giving ε-δ proofs of the following:
1. Let f be the function defined by
(
−8x + 10 if x ≤ 1,
f (x) =
3x − 1 if x > 1.

Show that lim f (x) = 2.


x→1

8
2. Let f be the function defined by
(
−4x + 5 if x < 1,
f (x) =
− 21 x + 32 if x > 1.

Show that lim f (x) = 1.


x→1

3. Let f be the function defined by




−8x − 2 if x < 0,
f (x) = −2 if x = 0,

1
7x − 2 if x > 0.

Show that lim f (x) = −2.


x→0

Test II around Wednesday, 16 November


Experienced teachers of Math 131 tell me that you will probably have a lot of
papers and the like due around the time of the test, and consequently will not
have a lot of time to study for it. Thus, I suggest you start studying now. You
may also want to think in terms of “practicing” rather than “studying”: redoing
old quiz and homework problems (without looking at the solutions, if you have
them, until afterwards) may be more helpful than simply reading over them.

9
Math 131, Lecture 19: The Chain Rule
Charles Staats
Wednesday, 9 November 2011

Important Note: There have been a few changes to Assignment 17. Don’t
use the version from Lecture 18.

1 Notes on the quiz


I’ve finally graded the quiz, and I wanted to make a few remarks.
1. Even apart from the one massive typo, the quiz directions were confusing.
I apologize for this. Next time (tomorrow), I will make sure to take more
time and care in designing the quiz.
Because the instructions were confusing, the quiz was difficult to grade.
Since the grades don’t actually count for anything, I did not try too hard
to assign grades based on what I thought was a “fair” assessment of your
work. Instead, I mostly concentrated on assigning grades—and making
comments—in whatever way I thought would be most useful to you.
2. In particular, on the delta-epsilon proof, assigning “fair” scores would have
been virtually impossible since the statement of the problem was incorrect.
Among other things, I deducted points for clear problems in the style of
the δ-ε proofs.
3. If δs and εs should appear in your solution, I will say so explicitly in
the problem instructions. In particular, if I ask you for the “limit-based
definition of the derivative,” I’m looking for something like

f (x + h) − f (x)
f 0 (x) = lim .
h→0 h
If I don’t ask you for εs and δs, don’t give them to me.

4. Do not—EVER—follow an expression like limh→0 by an equals sign:

f (x + h) − f (x)
f 0 (x) = lim =
h→0 h

1
This simply makes no sense. The notation limh→0 is supposed to represent
the limit of an expression. If you instead follow it by an equals sign, it’s
like saying “The limit of is. . . .”
If you write this on a test, I will deduct points.
5. When I ask you to give an ε-δ proof for a limit of a piecewise-linear
function, my secret goal is to get you to understand how you might go
about proving the following statement:
The two-sided limit exists and equals ` if and only if both the
one-sided limits exist and equal `.

Some of you gave separate ε-δ proofs of both the one-sided limits, and
then used the fact above to deduce the value of the two-sided limit. This
is perfectly correct, and would have received full credit if I were grading
the quiz for credit. However, since I was grading primarily to let you know
whether you are prepared for this sort of problem on the test, I deducted a
couple points. I very probably will give a problem like this on the test; if I
do so, I will explicitly state that you are not allowed to use the statement
above, so that I can justifiably deduct points if you do.
Having reread the sentence above, I realized it sounds like I am looking for
excuses to deduct points. This is NOT the case. When I give a particular
problem, there are certain things I am trying to see if you understand.
If you can do the problem correctly without understanding these things,
then my whole purpose in giving the problem is compromised.

2 Computing the derivative from the definition


There are several limit-based definitions of the derivative. They all amount to
saying “take the limit of the slope of the secant line between two points, as those
two points get close together:”

(x0 ,y0 )

2
The easiest definition to remember is probably
dy ∆y
= lim .
dx ∆x→0 ∆x
The easiest to compute with is probably based on the so-called “difference quo-
tient”:
dy f (x + h) − f (x)
= lim .
dx h→0 h
If I ask you to compute a derivative from the definition, I’m not mostly interested
in whether you can find the answer. I may even phrase the question something
like the following1 :
Use the definition of the derivative to prove that if f is the function
defined by f (x) = 1/x, then f 0 (x) = −1/x2 .
As you may note, I am in fact giving you the “answer” here: f 0 (x) = −1/x2 .
What I care about, when I ask a question like this, is whether you understand
the definition well enough to use it.
Although we’ve done this example in Lecture 17, I’m going to repeat it here,
since I will need it later in the lecture.
Example 1. Use the definition of the derivative to prove that if f is the function
defined by f (x) = x1 , then f 0 (x) = −1
x2 .

Solution.
f (x + h) − f (x)
f 0 (x) = lim
h→0 h
1 1
− x(x + h)
= lim x+h x ·
h→0 h x(x + h)
x − (x + h)
= lim
h→0 hx(x + h)
−h
= lim
h→0 hx(x + h)
−1
= lim
h→0 x(x + h)
−1
= 2.
x
If you want more examples of this sort of computation, you should review
Lecture 17. (There’s a version on Chalk that includes solutions to all of the
examples.)
1 NOTE: If I ask you this question on the test, the function f will be different.

3
3 Differentiating Quotients
We can use the Chain Rule together with the Product Rule and Example 1
(page 3) to differentiate quotients.
 
d 1
Example 2. Find .
dx x − 1
Recall, from Example 1, that Dx (1/x) = −1/x2 .

Solution.
 
d 1 −1 d
= · (x − 1)
dx x−1 (x − 1)2 dx
−1
= .
(x − 1)2

4 Proof Sketch of the Chain Rule


Let y be a function of x. What does it mean to say that the derivative of y at
x0 is equal to a number m?
∆y
lim =m
∆x→0 ∆x
∆y
⇐⇒ ∀ε > 0, ∃δ > 0 s.t. if 0 < |∆x| < δ, then −m <ε
∆x
|∆y − m∆x|
⇐⇒ ∀ε > 0, ∃δ > 0 s.t. if 0 < |∆x| < δ, then <ε
|∆x|
⇐⇒ ∀ε > 0, ∃δ > 0 s.t. if 0 < |∆x| < δ, then |∆y − m∆x| < ε|∆x|

Informally, this means that the statement m = f 0 (x) is equivalent to the state-
ment that
If the change in x is small, then ∆y ≈ m∆x.
In other words, near x0 , y is approximated by the tangent line. See Figure 1.
This suggests a (non-rigorous) definition of the derivative using infinitesi-
mals: if y = f (x), then f 0 (x) is the number such that

dy = f 0 (x) dx.

This “definition” is based on the general notion that “if something is approxi-
mately true for small ∆x, then it should be exactly true for dx because dx is so
small.” Thus, since ∆y ≈ f 0 (x)∆x, we get dy = f 0 (x) dx. This principle can get
you in big trouble if applied indiscriminately, which is why using infinitesimals

4
y
∆y = f 0 (x0 )∆x

y = f (x)

ε∆x

y0

x
x0 − δ x0 x0 + δ

|∆x| < δ

Figure 1: When ∆x is small, then ∆y is approximated by f 0 (x0 )∆x. In other


words, for ∆x small, the function is approximated by its tangent line (which
is defined by ∆y = f 0 (x0 )∆x). More precisely, the function is contained in a
narrow cone about the tangent line. The width of the cone is controlled by ε.
We can make the cone as narrow as we want (“arbitrarily narrow”), by making
δ (and hence ∆x) sufficiently small.

5
is “walking on clouds.” But in many circumstances, it can give good intuition
and correct results.
Now, suppose that y = f (u) and u = g(x), so that y = f (u) = f (g(x)).
Then we have dy = f 0 (u) du and du = g 0 (x) dx, so

dy = f 0 (u) du
= f 0 (u)g 0 (x) dx
= f 0 (g(x))g 0 (x) dx.

Hence, by the “infinitesimal definition of the derivative,”


dy
= f 0 (g(x))g 0 (x).
dx

Note: If I ask you on a test for the “Leibniz derivation of the Chain
Rule” or the “Infinitesimal derivation of the Chain Rule,” I am asking you,
more or less, to give me the paragraph above.

Theorem. (Chain Rule) If f and g are differentiable functions, then f ◦ g is


also differentiable, and

(f ◦ g)0 (x) = f 0 (g(x))g 0 (x).

The proof of the Chain Rule is to use εs and δs to say exactly what is meant
by “approximately equal” in the argument

∆y ≈ f 0 (u)∆u
≈ f 0 (u)g 0 (x)∆x
= f 0 (g(x))g 0 (x)∆x.

Unfortunately, there are two complications that have to be dealt with. The first
is that, for technical reasons, we need an ε-δ definition for the derivative that
allows |∆x| = 0. The following statement turns out to work:

∀ε > 0, ∃δ > 0 s.t. if |∆x| < δ, then |∆y − f 0 (x0 )∆x| ≤ ε|∆x|.

Comparing this to the earlier version, we got rid if the requirement 0 < |∆x|
by changing the final < ε|∆x| to ≤ ε|∆x|. I don’t want to explain why exactly
we can do this, but anyone who has taken (and understood) an analysis course
ought to be able to do it without much trouble.
The second complication is that the expression for δ in terms of ε turns
out to be a bit ugly. For this reason, I will spare you the details. However, I
hope I have convinced you that the basic idea of the proof of the Chain Rule is
comprehensible, even if the technical details are a bit involved.

6
Assignment 17 (due Friday, 11 November)
From Section 2.3:
• Problems 5–8. Do each problem two ways—using the limit definition of
your choice, and using the rules of differentiation (including the Chain
Rule, if you find it helpful).
• Problems 17–20.

• Problems 31–32. Do not FOIL out the products; instead, use the product
rule for differentiation.
The even-numbered problems will be graded carefully.

Section 2.5, Problems 1–4. Make sure it is clear, from your answer, how you are
using the Chain Rule (see, for instance, Example 3 at the end of Lecture 18).
Problems 2 and 4 will be graded carefully.

Give an ε-δ proof for each of the following. Do not use the fact that if both the
one-sided limits exist and are equal, then the two-sided limit exists and is equal
to both of them.
1. Let f be the function defined by
(
7x − 3 if x ≤ 0,
f (x) =
− 91 x − 3 if x > 0.

Show that lim f (x) = −3.


x→0

2. Let f be the function defined by


(
−1x − 18
if x < 3,
f (x) = 1 7 7 7

6x − 2 if x ≥ 3.

Show that lim f (x) = −3.


x→3

3. Let f be the function defined by



 1 3
− 2 x + 2 if x < −3,
f (x) = 4 if x = −3,


3x + 12 if x > −3.

Show that lim f (x) = 3.


x→−3

7
Problems 1 and 3 will be graded carefully.

[NOTE: This is now a Bonus Problem.] Suppose y = f (x) and f (x0 ) = y0 .


A purely ε-δ version of the statement that “f is continuous at x0 ” is given as
follows:
∀ε > 0, ∃δ > 0 s.t. if |x − x0 | < δ, then |y − y0 | < ε.
Use this definition to prove the following fact:

Suppose that

u = f (x),
u0 = f (x0 ),

y = g(u) = g(f (x)), and


y0 = g(u0 ) = g(f (x0 )).

If f is continuous at x0 and g is continuous at u0 , then g ◦ f is


continuous at x0 .

Assignment 18 (due Monday, 14 November


Section 2.2, Problems 9, 10, 15, and 16. These problems are about the process
of computing the derivative from the limit; finding the “answer” by another
method will not receive full credit. You do not need to hand in any of these,
but a similar problem will appear on the test.

Section 2.3, Problems 27–30. Use the Product Rule. Problems 28 and 30 will
be graded carefully.

Section 2.5, Problems 5–8, 13–14, and 17–18. You do not need to show every
single step, but it should be clear to the grader how you got to the answer. The
even-numbered problems will be graded carefully.

Differentiate the following expressions with respect to x. (Hint: Apply the


Chain Rule more than once.) You do not need to show every single step, but it
should be clear to the grader how you got to the answer. You do not need to
simplify the answer.
42
1. 5(2x + 1)361 − 17
1776
2. 1 − (1 − 2x)33
Both of these will be graded carefully. Since a similar problem may appear on
the test, and this homework set will almost certainly not be graded before the
test, you may want to ask to go over the answers in tutorial on Tuesday.

8
Give ε-δ proofs of the following facts. Do not use the fact that if both the
one-sided limits exist and are equal, then the two-sided limit exists and is equal
to both of them.
1. Let f be the function defined by
(
−8x + 10 if x ≤ 1,
f (x) =
3x − 1 if x > 1.

Show that lim f (x) = 2.


x→1

2. Let f be the function defined by


(
−4x + 5 if x < 1,
f (x) =
− 21 x + 32 if x > 1.

Show that lim f (x) = 1.


x→1

3. Let f be the function defined by




−8x − 2 if x < 0,
f (x) = −2 if x = 0,

1
7x − 2 if x > 0.

Show that lim f (x) = −2.


x→0

Problems 1 and 3 will be graded carefully.

Test II around Wednesday, 16 November


Experienced teachers of Math 131 tell me that you will probably have a lot of
papers and the like due around the time of the test, and consequently will not
have a lot of time to study for it. Thus, I suggest you start studying now. You
may also want to think in terms of “practicing” rather than “studying”: redoing
old quiz and homework problems (without looking at the solutions, if you have
them, until afterwards) may be more helpful than simply reading over them.

9
Math 131, Lecture 20: The Chain Rule, continued
Charles Staats
Friday, 11 November 2011

1 A couple notes on quizzes


I have a couple more notes inspired by the quizzes.

1.1 Concerning δ-ε proofs


First, concerning δ-ε proofs. There are a couple of commonsense rules that can
alert you to when you are making a very bad choice of δ.
First, δ is always a positive number. If you ever find yourself writing some-
thing like δ = − 21 ε, you should immediately realize you’ve done something
wrong. The most way to get here was to forget the absolute value signs around
the −2 when writing something like
|−2(x − 1)| = |−2| · |x − 1|.
Second, δ is typically a very small positive number. If you write something
like δ = 2 + ε, then there is no possibility that δ will ever be less than 2. This
is almost never what you want.
An error like demonstrates clear limits to your understanding of ε-δ proofs,
and will probably cost you more points on a test than a less obvious error.

1.2 Infinitesimals are never equal to finite quantities


A finite quantity is something like 1+x without any d’s in it. A ratio of infinites-
imals like dy/dx is also a finite quantity. But if you multiply by a d(something)
and don’t divide by one, then your quantity is infinitesimal. You can equate two
infinitesimals (e.g., dy = f 0 (x) dx) or two finite quantities (e.g., dy/dx = f 0 (x)).
But if you ever have a finite quantity equal to an infinitesimal quantity, then
you are doing something terribly wrong. In particular, for formulations of the
product rule, we have
d dv du
(uv) = u +v X
dx dx dx
d(uv) = u dv + v du X
d
(uv) = u dv + v du
dx

1
y
∆y = f 0 (x0 )∆x

y = f (x)

ε∆x

y0

x
x0 − δ x0 x0 + δ

|∆x| < δ

Figure 1: When ∆x is small, then ∆y is approximated by f 0 (x0 )∆x. In other


words, for ∆x small, the function is approximated by its tangent line (which
is defined by ∆y = f 0 (x0 )∆x). More precisely, the function is contained in a
narrow cone about the tangent line. The width of the cone is controlled by ε.
We can make the cone as narrow as we want (“arbitrarily narrow”), by making
δ (and hence ∆x) sufficiently small.

2
2 The Infinitesimal derivation of the Chain Rule
As you may recall from last lecture, the infinitesimal derivation of the Chain
Rule goes something like this:
Let y = f (u) and u = g(x). Then we have
dy = f 0 (u) |{z}
du
z }| {
= f 0 (|{z}
u ) g 0 (x) dx, since du = g 0 (x)dx
z}|{ 
= f 0 g(x) g 0 (x) dx since u = g(x).
Hence,
dy
= f 0 (g(x))g 0 (x), i.e.,
dx
(f ◦ g)0 (x) = f 0 (g(x))g 0 (x).

The “infinitesimal statement” that dy = f 0 (u) du corresponds to the “approx-


imate statement” that ∆y ≈ f 0 (u)∆u. The basic idea behind the proof of
the Chain Rule is to come up with a precise, ε-δ version of this “approximate
statement,” and then use that to turn the notion that
∆y ≈ f 0 (u)∆u ≈ f 0 (u)g 0 (x)∆x
into a precise proof. I will not repeat this ε-δ statement here, but I have included
an illustration of it for your viewing pleasure in Figure 1.

3 Remembering the Chain Rule


Recall the Chain Rule, as stated in the last lecture:
Theorem. If f and g are differentiable functions, then
(f ◦ g)0 (x) = f 0 (g(x)) · g 0 (x).
This is probably the form in which the Chain Rule is easiest to use, but it’s
kind of hard to remember. It becomes a lot easier to remember if we restate it in
Leibniz notation. To do this, assume that u = g(x) and y = f (u) = f (g(x)) =
(f ◦ g)(x). Thus, we have
dy
(f ◦ g)0 (x0 ) =
dx x=x0

0 0 dy dy
f (g(x0 ) = f (u0 ) = =
du u=u0 du u=g(x0 )
du
g 0 (x0 ) =
dx x=x0

3
and the Chain Rule becomes
! !
dy dy du
= ,
dx x=x0 du u=g(x0 ) dx x=x0

or more simply,
dy dy du
= · .
dx du dx
When written this way, the Chain rule seems completely obvious—just cancel
the du’s. This is not a great way to think about why the Chain Rule is actually
true, because unlike most infinitesimal arguments, it cannot be turned into a
rigorous proof. If I ask you for the infinitesimal or Leibniz derivation
of the Chain Rule on the test, the explanation here will not receive
full credit. However, it does make a good mnemonic device.

4 Using the Chain Rule


The textbook’s section on the Chain Rule (Section 2.5) is actually not bad, and
you might want to take a look at it (especially if you find my notes confusing).
To quote the textbook, the key idea in applying the Chain Rule is that
The last step in calculation corresponds to the first step in differen-
tiation.
Example 1. Use the Chain Rule to differentiate (2x + 1)3 .
Again quoting the textbook (more or less), the last step in the calculation is to
cube something, so you start off by differentiating the cube function.
Solution (long version). Let

u = 2x + 1
y = u3 = (2x + 1)3 .

Then
du
=2
dx
dy
= 3u2
du
and so
dy dy du
=
dx du dx
= 3u2 · 2
= 6(2x + 1)2 ,

where the last step is obtained by substituting in u = 2x + 1.

4
Solution (short version).

d d
(2x + 1)3 = 3(2x + 1)2 · (2x + 1)
dx dx
2
= 3(2x + 1) · 2
= 6(2x + 1)2 .

5 Differentiating Quotients
Recall that last lecture, we computed that
d 1 −1
= 2.
dx x x
We can use this, together with the Chain Rule, to compute a lot of derivatives.
To start with, we will take a look at xn when n is a negative integer.
Example 2. Let f (x) = x−m , where m is a positive integer. We may use the
Chain Rule to compute f 0 (x), as follows:
 
df d 1
=
dx dx xm
−1 d m
= m 2· (x )
(x ) dx
−1
= 2m · mxm−1
x
= −m · x−2m+(m−1)
= −m · x−m−1 .

If n = −m is a negative integer, then we get


d n
x = nxn−1 .
dx
Thus, we have that the power rule holds for negative integers as well as
positive integers. Since Dx (x0 ) = Dx (1) = 0 = 0 · x−1 for x 6= 0, it might also
be said that the power rule holds for 0.
Theorem. (Power Rule—all integers) If n is an integer, then
d n
(x ) = nxn−1 .
dx
This should probably be memorized, but if you already have the one for
positive integers memorized, that will probably not be difficult. (We will later

5
show using implicit differentiation that the Power Rule holds whenever n is a
rational number. It is in fact true even when n is irrational, although proving
that requires logarithms.)
We can also use the Chain Rule, together with the Product Rule, to differ-
entiate quotients.

Theorem. (Quotient Rule) Let f and g be differentiable functions. Then


 
f (x) g(x)Dx (f (x)) − f (x)Dx (g(x))
Dx =
g(x) g(x)2
g(x)f (x) − f (x)g 0 (x)
0
= .
g(x)2

You can either memorize the Quotient Rule, or remember how to differentiate
quotients by combining the Product Rule with the Chain Rule. As long as you
can differentiate quotients, I don’t much care which method you use. If you do
want to memorize this, the standard mnemonic is
“Dee quotient equals bottom Dee top minus top Dee bottom, all
over bottom squared.”
However, if you use this mnemonic, remember not to equate infinitesimals with
finite quantities. Either all the Dees should be Dx (derivative with respect to
x, a finite quantity) or they should all be d (gives infinitesimals on both sides).
Proof.
   
f (x) 1
Dx = Dx f (x) ·
g(x) g(x)
 
1 1
= f (x)Dx + Dx (f (x)) (product rule)
g(x) g(x)
−1 f 0 (x)
= f (x) · 2
· Dx (g(x)) + (chain rule)
(g(x)) g(x)
0 0
−f (x)g (x) f (x)
= +
g(x)2 g(x)
−f (x)g (x) + f 0 (x)g(x)
0
=
g(x)2
g(x)f 0 (x) − f (x)g 0 (x)
= .
g(x)2

6
Example 3. Let
x+1
f (x) = .
x−1
Compute f 0 (x).

Without the quotient rule.


   
d x+1 d 1
= (x + 1) ·
dx x − 1 dx x−1
 
d 1
= (x + 1)
dx x − 1
1 d
+ (x + 1) (Product Rule)
x − 1 dx
−1 d
= (x + 1) · · (x − 1)
(x − 1)2 dx
1
+ ·1 (Chain Rule)
x−1
x+1 1
=− +
(x − 1)2 x−1
−(x + 1) + (x − 1)
=
(x − 1)2
−2
= .
(x − 1)2

With the quotient rule.


 
x+1 (x − 1) · 1 − (x + 1) · 1
Dx =
x−1 (x − 1)2
x−1−x−1
=
(x − 1)2
−2
=
(x − 1)2

7
Assignment 18 (due Monday, 14 November)
Section 2.2, Problems 9, 10, 15, and 16. These problems are about the process
of computing the derivative from the limit; finding the “answer” by another
method will not receive full credit. You do not need to hand in any of these,
but a similar problem will appear on the test.

Section 2.3, Problems 27–30. Use the Product Rule. Problems 28 and 30 will
be graded carefully.

Section 2.5, Problems 5–8, 13–14, and 17–18. You do not need to show every
single step, but it should be clear to the grader how you got to the answer. The
even-numbered problems will be graded carefully.

Differentiate the following expressions with respect to x. (Hint: Apply the


Chain Rule more than once.) You do not need to show every single step, but it
should be clear to the grader how you got to the answer. You do not need to
simplify the answer.
42
1. 5(2x + 1)361 − 17
1776
2. 1 − (1 − 2x)33
Both of these will be graded carefully. Since a similar problem may appear on
the test, and this homework set will almost certainly not be graded before the
test, you may want to ask to go over the answers in tutorial on Tuesday.

Give ε-δ proofs of the following facts. Do not use the fact that if both the
one-sided limits exist and are equal, then the two-sided limit exists and is equal
to both of them.

1. Let f be the function defined by


(
−8x + 10 if x ≤ 1,
f (x) =
3x − 1 if x > 1.

Show that lim f (x) = 2.


x→1

2. Let f be the function defined by


(
−4x + 5 if x < 1,
f (x) =
− 21 x + 32 if x > 1.

Show that lim f (x) = 1.


x→1

8
3. Let f be the function defined by


−8x − 2 if x < 0,
f (x) = −2 if x = 0,

1
7x − 2 if x > 0.

Show that lim f (x) = −2.


x→0

Problems 1 and 3 will be graded carefully.

Test II around Wednesday, 16 November


Experienced teachers of Math 131 tell me that you will probably have a lot of
papers and the like due around the time of the test, and consequently will not
have a lot of time to study for it. Thus, I suggest you start studying now. You
may also want to think in terms of “practicing” rather than “studying”: redoing
old quiz and homework problems (without looking at the solutions, if you have
them, until afterwards) may be more helpful than simply reading over them.

Assignment 19 (due Friday, 18 November)


Section 2.6, Problems 1–4. Problems 2 and 4 will be graded carefully.

Assume that f is a differentiable function. Consider the two functions g and h


defined by

g(t) = 2f (t),
h(t) = f (2t).

You may want to check your answers below by considering the specific cases of
f (t) = t and f (t) = t2 .
1. Explain how to obtain the graphs of g and h from the graph of f by
shrinking/stretching.
2. Compute g 0 and h0 in terms of f 0 . (Hint: they are NOT the same.)

3. Explain how to obtain the graphs of g 0 and h0 from the graph of f 0 by


shrinking/stretching.
All three of these will all be graded carefully.

9
Math 131, Lecture 21
Charles Staats
Monday, 14 November 2011

1 Differentiability and Continuity


There’s a theoretical point that I’ve sort of hand-waved over up to now, but that
probably needs to be addressed. If you recall, the definition of the derivative
(or at least, one of the definitions) is
f (x0 + h) − f (x0 )
f 0 (x0 ) = lim .
h→0 h
However, an important point about limits is that they don’t always exist. Sim-
ilarly, derivatives do not always exist.
Example 1. Let f be the absolute value function; i.e., f is defined by
(
−x if x < 0,
f (x) = |x| =
x if x ≥ 0.
Then f 0 (0) does not exist. Geometrically, we can see this because it is not
possible to draw a narrow cone centered on (0, 0) that contains the graph of f :
f (x)

x
−4 −3 −2 −1 1 2 3 4
−1

−2

−3

−4

1
More formally, we see that
|0 + h| − |0| −h
lim− = lim = −1
h→0 h h→0 h
|0 + h| − |0| h
lim = lim = 1.
h→0+ h h→0 h

Since the one-sided limits are not equal, the two-sided limit
|0 + h| − |0|
lim = f 0 (0)
h→0 h
does not exist.
Definition. We say that a function f is differentiable at x0 if f is defined at x0
and the derivative f 0 (x0 ) exists (and is finite). We say that f is differentiable if
it is differentiable at every point of its domain.

Example 2. Consider the function f defined by f (x) = 3 x:

f (x)

x
−4 −3 −2 −1 1 2 3 4
−1

−2

−3

−4

You should not find it hard to believe that the tangent line to f at the origin is
vertical—i.e., a line with slope infinity. Correspondingly, if one evaluates

(0 + h)1/3 − 01/3
f 0 (0) = lim ,
h→0 h
one will find that the limit is ∞. Since this derivative is not finite, we still say
that f is not differentiable at 0.
One reason for this convention is that the pChain Rule does not work here: if
it did, it would tell us that the derivative of 3 g(x) at x = 0 is ∞·g 0 (0). Say that

2
p
g(x) = x3 ; then we know that 3 g(x) = x, so the derivative at 0 should be 1;
but the chain rule would tell us that this derivative is ∞·0, which does not make
sense. However, because
√ the Chain Rule only applies when both functions are
differentiable, and 3 is not differentiable, we don’t run into a contradiction.
As I have said before, the “main point” of functions is, more or less, that
they give us a way to talk about things that we don’t have formulas for. Thus,
if we have a problem, we might be able to show that there is a function that
gives its solution, even if there is no formula for the solution. Once we’ve shown
that the solution is given by a function, we can ask how “nice” the function is:
Is it continuous? Is it differentiable? In this situation, we would probably find
the following theorem very interesting:
Theorem. Let f be a function. If f is differentiable at x0 , then f is continuous
at x0 .
Proof. Assume f is differentiable at x0 . Then f is defined at x0 , by definition
of differentiability.
Let y = f (x), y0 = f (x0 ), and note that
y = y0 + ∆y
∆y
= y0 + · ∆x.
∆x
Hence,
lim f (x) = lim y
x→x0 ∆x→0
∆y
= lim y0 + · ∆x
∆x→0
 ∆x 
∆y  
= y0 + lim · lim ∆x
∆x→0 ∆x ∆x→0
!
dy
= y0 + ·0
dx x=x0
= y0 = f (x0 ).
Note: In the proof above, if the dy/dx|x=x0 did not exist (as a finite number),
then the limit
∆y
lim
∆x→0 ∆x
would not have made sense, and so the Main Limit Theorem would not have
been applicable.
For our purposes in this class, the most important use of this theorem may
be a way to tell when a function is not differentiable. For this, we use the
contrapositive:

If f is discontinuous at x0 , then f is not differentiable at x0 .

3
Warning. It is quite possible for a function to be continuous but not
√ differen-
tiable. For instance, our earlier examples f (x) = |x| and g(x) = 3 x are both
continuous, but neither is differentiable at 0.

2 Higher derivatives
Given a differentiable function f , its derivative f 0 is also a function. This
function f 0 may itself be differentiable, and have a derivative of its own, which
we call f 00 —the second derivative. The derivative of f 00 , if it exists, is denoted
f 000 , and called the third derivative of f .
Example 3. Let f be the function defined by f (x) = x5 . Find the first, second,
and third derivatives of f .

Solution.

f 0 (x) = 5x4
f 00 (x) = 5 · 4x3 = 20x3
f 000 (x) = 20 · 3x2 = 60x2 .

We can, of course, proceed to take higher derivatives than just the third deriva-
tive. But since something like f 0000000 (x) would be rather hard to read, we denote,
e.g., the seventh derivative of f by f (7) . There are several other notations:
read aloud prime notation D notation Leibniz notation
dn f
the nth derivative of f (n) (x) Dxn (f (x))
dxn
f with respect to x
 
d dy
The Leibniz notation is based on the idea that should be written
dx dx
d2 y
as . Unlike most other versions of the Leibniz notation, this is purely a
dx2
mnemonic device; trying to think about this as the “quotient” of “infinitesimal”
quantities d2 y and dx2 ends up just giving a mess.

4
Test Wednesday, 16 November
Test II will be on Wednesday. It will cover the lectures up through Lecture 20
(Friday), and the homework up through Assignment 18 (due today). You should
also look at the quizzes (graded and ungraded) up to and including tomorrow’s
quiz. Although there will be one problem involving limits and ε-δ proofs, the
emphasis of the test will be on derivatives, rather than limits. (Note, however,
that you will be asked to find a derivative from the definition, which will require
you to evaluate a limit.)
If you can do all the quiz problems without consulting anyone or anything,
you should do well. When you’ve mastered the quiz problems, there still may be
some additional benefit from practicing homework problems. Bonus problems
will not be tested.
Last Tuesday, I sent out an e-mail outlining what sorts of problems I was
thinking about putting on the test. As promised, that list is not perfect—some
of the problems their did not make it, and one or two things that ended up on
the test were not on the list. However, it’s still a fairly good study guide.
The lecture notes, in addition to trying to explain the material, discuss some
pitfalls that could cost you points, even if you think you know what you are
doing.
On the last test, a lot of people who got basically the right answers still lost
points, because what they wrote did not show me that they truly understood
what was going on. If you want to know exactly how much you need to write
to get credit on certain kinds of problems, please come to office hours (Tuesday,
3–4pm) and ask me. If you cannot make my office hours, I’ll be happy to make
an appointment with you.

Assignment 19 (due Friday, 18 November)


Section 2.6, Problems 1–4. Problems 2 and 4 will be graded carefully.

Assume that f is a differentiable function. Consider the two functions g and h


defined by
g(t) = 2f (t),
h(t) = f (2t).
You may want to check your answers below by considering the specific cases of
f (t) = t and f (t) = t2 .
1. Explain how to obtain the graphs of g and h from the graph of f by
shrinking/stretching.
2. Compute g 0 and h0 in terms of f 0 . (Hint: they are NOT the same.)
3. Explain how to obtain the graphs of g 0 and h0 from the graph of f 0 by
shrinking/stretching.
All three of these will all be graded carefully.

5
Math 131, Lecture 22
Charles Staats
Friday, 18 November 2011

Organizational matters
• Tutorial scheduling for next quarter

• Vote: Do you want an assignment due Wednesday, November 23 (the day


before Thanksgiving), or a double assignment due Monday, November 28?

1 Higher derivatives
Given a differentiable function f , its derivative f 0 is also a function. This
function f 0 may itself be differentiable, and have a derivative of its own, which
we call f 00 —the second derivative. The derivative of f 00 , if it exists, is denoted
f 000 , and called the third derivative of f .
Example 1. Let f be the function defined by f (x) = x5 . Find the first, second,
and third derivatives of f .

Solution.

f 0 (x) = 5x4
f 00 (x) = 5 · 4x3 = 20x3
f 000 (x) = 20 · 3x2 = 60x2 .

We can, of course, proceed to take higher derivatives than just the third deriva-
tive. But since something like f 0000000 (x) would be rather hard to read, we denote,
e.g., the seventh derivative of f by f (7) . There are several other notations:
read aloud prime notation D notation Leibniz notation
dn f
the nth derivative of f (n) (x) Dxn (f (x))
dxn
f with respect to x

1
 
d dy
The Leibniz notation is based on the idea that should be written
dx dx
d2 y
as . Unlike most other versions of the Leibniz notation, this is purely a
dx2
mnemonic device; trying to think about this as the “quotient” of “infinitesimal”
quantities d2 y and dx2 ends up just giving a mess.

2 Acceleration
If you recall from when we first introduced the derivative, the first motivation
I gave was that “the derivative is the rate of change of position with respect to
time.” If x represents position, then
dx
dt
represents velocity. One of the most important instances of a higher derivative
is acceleration, or the rate of change of velocity with respect to time:

d2 x
.
dt2
If the velocity of an object is increasing, then its acceleration is positive; if the
velocity is decreasing, then the acceleration is negative.
Intuitively, we are inclined to think that something is “accelerating” if it is
“getting faster,” and “decelerating” if it is ”slowing down.” This intuition can
be useful, but it is also dangerous. If an object has positive velocity (i.e., moving
to the right), but negative acceleration, then its velocity will decrease to zero,
and continue to decrease to be negative; i.e., the object will start moving to the
left. We could say that the object decelerates to a stop, and then accelerates
in the opposite direction; however, this is deceptive, because the (negative)
acceleration is exactly the same before, during, and after the instant at which
the object is “stopped.”
For another example, consider what happens when an object is tossed up-
wards. We might be inclined to say that under the force of gravity, it decelerates
until it reaches the apex of its path, and then starts falling downward. But re-
ally, the acceleration is the same (negative) from the moment the object leaves
the hand. Thus, it actually makes more sense to say that the object is falling
from the instant it leaves the hand—even while it is still moving upward (i.e.,
has positive velocity).
One time (in middle school, I think), I was in an auditorium with a bunch of
other students listening to an astronaut speak. At one point, he asked us why,
when an astronaut in a spaceship “drops” something, it floats rather than falling.
The auditorium shook as everyone in the audience shouted, “No gravity!” The
astronaut replied, “Everyone who just said ‘no gravity’ is 100% wrong.” If there
were no gravity, then the spaceship would not be orbiting the earth; instead, it
would be traveling away from the earth in a straight line, never to return. The

2
without gravity
ground level

Figure 1: The spaceship is falling, but it’s moving sidewise so quickly that by
the time it reaches “ground level,” it has not actually gotten any closer to the
surface of the earth.

reason, he said, that an object dropped inside the spaceship appears to float
is that the object, the astronaut, and the entire spaceship are already falling.
The only reason the spaceship does not reach the ground is that by the time
it reaches “ground level,” it’s moved so far horizontally that the ground has
dropped out from beneath it.
For trajectories short enough that we can pretend the earth is flat, the
general rule is the following:
Law of Falling Bodies. If an object is under no influences1 but that of gravity,
then its vertical acceleration is a constant g ≈ −10 sm2 .
If the acceleration g is measured in feet per second squared rather than
meters per second squared, its value is approximately −32. In either case, this
acceleration is negative, because the object’s velocity is decreasing. (If the object
is moving downward, then its velocity is already negative, and is becoming more
negative.)
I’ve called this the Law of Falling Bodies rather than the Law of Gravity
because gravity generally refers to a deeper phenomenon discovered by Isaac
Newton, whereas a version of the Law of Falling Bodies was known earlier to
Galileo. (Who was forced to deal with average acceleration, because he did not
know about derivatives.)
If we translate the Law of Falling Bodies into mathematical notation, we
obtain the equation
d2 y
= −10,
dt2
where y is the vertical position of the object. This is a very simple example
of what is called a differential equation; to “solve” the differential equation, we
1 If this were a physics course, we’d use the word “forces.”

3
what functions y(t) would make it true. It is not hard to verify that for any
choice of a and b, the function

y(t) = −5t2 + at + b

is a solution to the differential equation:

y 0 (t) = −10t + a
y 00 (t) = −10.

As it turns out, these are the only functions that satisfy this differential equation,
although we will not see why until next quarter. Thus, any time you toss or
drop an object, its vertical position is described by

y = −5t2 + at + b

for some choice of a and b. Note that there are many different paths possible,
since there are many different values of a and b. This is good, since there are
many different paths falling bodies can follow in real life. (If you throw a piece
of chalk up, it will follow a different path from the piece of chalk you throw
down, but both paths can be described by the equation y = −5t2 + at + b, for
some (different) values of a and b.)
However, no matter what a and b are, y = −5t2 + at + b is always some
sort of upside-down parabola. Correspondingly, a falling object always moves
in some form of upside-down parabola; see Figure 2a. If you imagine that your
object is a droplet of water, and you string a bunch of these “objects” together
in a continuous stream, you can see the whole path at once, as in Figure 2b.
Notice how much more interesting nature’s parabola is than the stark, abstract
curve given in 2a. The water’s arc seem to scintillate with reflected light; cords
of water seem to twist together, like the muscles in a Michelangelo drawing of
an arm.
In the textbook, it essentially just gives you the equation for the position,
say y = −5t2 + t + 1, and asks you to calculate the acceleration. And as it turns
out, the acceleration is constantly −10 (or perhaps −32, since the textbook
seems to like feet more than meters). While finding the acceleration from the
position function is a perfectly good exercise, it somehow feels backwards. In
some sense, the basic statement is that the vertical acceleration of a falling
object is constantly −10; this basic fact is the cause of the effect that the object
travels in a parabola given by y = −5t2 + at + b. By starting off with the path
and deducing the acceleration, it feels as though you are mixing up the cause
and the effect.
One final note: If you look more closely at Figure 2a, you will see that the
horizontal axis is indicating time. On the other hand, in Figure 2b, the “hor-
izontal axis,” such as it is, clearly is given by horizontal position, or distance
(more or less). Since the graph does not actually tell you where the object is
horizontally at a given time, it is not entirely clear why the “parabola” descrip-
tion should be accurate; the graph could just as easily describe an object that

4
y

(a) An object with constant negative accelera-(b) Parabolic trajectory of water. By GuidoB.
tion moves in an upside-down parabola. Modified (primarily to make it grayscale).
This image is licensed under a Creative
Commons Attribution-Share Alike 3.0 Un-
ported license; see http://creativecommons.
org/licenses/by-sa/3.0/deed.en.

Figure 2: Parabolas in theory and in practice

goes straight up and straight back down with no “sideways” movement. For the
moment, I’m just going to ignore this discrepancy. We may, or may not, discuss
it when discussing related rates.

3 Implicit Differentiation
Suppose we know, or suspect, that y is a differentiable function of x. We don’t
have a formula for y, but we may know that y and x satisfy some relation, for
instance,
y 2 + x2 = 1.
Often, we can use this, together with the chain rule, to figure out what the
derivative of y must be (assuming it has one). In the example at hand, we

5
differentiate both sides with respect to x, and then solve for the derivative Dx y:

Dx (y 2 + x2 ) = Dx (1)
2y · Dx (y) + 2x = 0
2yDx y = −2x
−2x −x
Dx y = = .
2y y
This expression for the derivative Dx y has a y in it as well as an x, which, as the
book says, can be “a nuisance.” However, it can nevertheless be quite useful. If
we should happen to know that the value of y at a point x0 is y0 , then we can
use this to calculate Dx y = dy/dx at the point (x0 , y0 ), assuming this derivative
exists.

6
Assignment 20 (due Monday, 21 November)
Section 2.3, Problems 39–40. Problem 40 will be graded carefully.

Section 2.6, Problems 7–10. Problems 8 and 10 will be graded carefully.

Section 2.7, Problems 1–2. Your expression for Dx y may include both xs and
ys. Problem 2 will be graded carefully.

The following argument purports to show that every function is continuous:


Let f be a function, and x0 any point in its domain. We show
that f is continuous at x0 .

lim f (x) = lim f (x0 ) + [f (x) − f (x0 )]


x→x0 x→x0
f (x) − f (x0 )
= lim f (x0 ) + · (x − x0 )
x→x0 x − x0
   
f (x) − f (x0 )
= f (x0 ) + lim · lim x − x0
x→x0 x − x0 x→x0
 
f (x) − f (x0 )
= f (x0 ) + lim ·0
x→x0 x − x0
= f (x0 ) + 0,

since anything times zero is zero. Thus, f is continuous at x0 . More-


over, since the same argument applies to every point x0 in the do-
main of f , we know that f is continuous at every point in its domain.
In other words, f is continuous.

On the other hand, we know that not every function is continuous. Thus, there
must be a flaw in the argument. What is it? (Hint: this argument can be used
to show that every differentiable function is continuous.)

Part(?) of Assignment 21 (due Wednesday, 23


November)
[Note: If you won’t be in class on the day before Thanksgiving, then some time
before class, put the homework in my mailbox (in the Eckhart basement). Also,
send me an email so that I know you have done this.]

Section 2.6, Problems 11–12 and 20–21. Problems 12 and 21 will be graded
carefully.

Section 2.7, Problems 3–6 and 19–20. The even-numbered problems will be
graded carefully.

7
(“Semi-bonus problem”) Suppose that

lim f (x) = ` = lim f (x);


x→c− x→c+

i.e., both the one-sided limits are defined, and they are equal. Use the ε-δ
definition of the limit to show that the two-sided limit is also defined and equal
to `, i.e., that
lim f (x) = `.
x→c

The technique involved should be similar to that used to give ε-δ proofs for
piecewise linear functions.
This is a “semi-bonus problem” in the following sense:
• If you do not seriously attempt it, you will not receive full credit on their
homework.

• If you seriously attempt it, you will receive full credit for it (although your
actual homework grade will, of course, depend on the other homework
problems).
• If you get it right, you will receive a bonus point on the homework.

8
Math 131, Lecture 23
Charles Staats
Monday, 21 November 2011

1 Implicit Differentiation: How to differentiate


a function we don’t know
All of the “exercises” for differentiation so far have been based on differentiating
formulas. However, many of the rules for differentiation (most especially, the
chain rule) are much more general than this: they deal with differentiating
functions. And, as you may recall, kind of the whole point of functions is that
they are not necessarily given by formulas. We have not really explored this
very far, because most of the functions we could talk about were, in fact, given
by formulas. But there is another √ way: we can define a function as a solution
to something. For instance, the function is really defined by

x = the nonnegative number y such that y 2 = x.

In other words, x is just a fancy way of writing “the (nonnegative) solution to
the equation y 2 = x.” And we can go back to this basic definition to differentiate
the square root function:

Example 1. Suppose y = x. Find an expression for dy/dx.

Solution. We assume, first of all, that x is in fact differentiable; without this
assumption, there is not much we can do. We then go back to the basic equation
that y 2 = x and apply the Chain Rule:
y2 = x
d 2 d
y = x
dx dx
dy
2y · =1
dx
dy 1 1
= = √ .
dx 2y 2 x

Thus, assuming that the function f taking x 7→ x is differentiable, its deriva-
tive f 0 is necessarily given by
1
f 0 (x) = √ .
2 x

1

Exercise 2. Use implicit differentiation to show that if y = − x, then

dy 1
=− √ .
dx 2 x

√ 2
Solution. This time, y also satisfies the equation y 2 = x, since (− x) =
√ 2
(−1)2 ( x) = 1 · x = x. Thus, we can differentiate implicitly:

y2 = x
dy
2y · =1
dx
dy 1 1 1
= = √ =− √ .
dx 2y 2 (− x) 2 x

Note that there √ is a problem we never dealt with here: we never actually
showed that f (x) = x is differentiable. We only figured out what its derivative
must be, if the derivative exists. There are a couple ways to solve this problem.

• It is possible to compute the derivative of x directly from the defini-
tion of the derivative (i.e., as the limit of the difference quotient). This
is done earlier in the textbook. However, this is a way to avoid using
implicit differentiation; what we really want is a way to show that implicit
differentiation works.
• There is a theorem called the “Implicit Function Theorem” that states,
roughly, that if implicit differentiation gives a reasonable answer, then the
equation in question does in fact have a solution y = f (x) where f is a
differentiable function. This is kind of like the Main Limit Theorem: If
the process gives a reasonable answer, then we know that must be the
right answer; but if the process does not give a reasonable answer, we
don’t know anything.
The Implicit Function Theorem may seem to be the answer to our problems,
but there are subtleties even here. First, the actual statement of the theorem is
something that I find confusing, so I very much doubt that you want to see it.
Second, while the Implicit Function Theorem
√ can guarantee√ that some solutions
are differentiable (in this case, f (x) = x and f (x) = − x are both solutions
to f (x)2 = x that are differentiable for x > 0), there will also be other solutions
that are not differentiable. For instance, if f is the function defined by
(√
x if 0 < x ≤ 1,
f (x) = √
− x if x > 1,

2
f (x)

x
−1 1 2 3 4

−1

−2

−3

then y = f (x) is also a solution to the equation y 2 = x for all x > 0, but f is
not even continuous, much less differentiable. We will not try to explain why
the Implicit Function Theorem applies for some “solutions,” but not to others.
Instead, we will adopt a “third way”:
• Ignore the difficulties and just assume implicit differentiation works. Any
function we encounter “naturally” in this course1 is going to work out just
fine.

In essence, we’ve reached a point where the skyscraper just gets too convoluted
to deal with, so we’re going to continue walking on clouds.
There’s one more very important result we want to obtain using implicit dif-
ferentiation. Recall that we proved the Power Rule, Dx (xn ) = nxn−1 , whenever
n is an integer. We’re now going to that this holds, not just for integers, but
for rational numbers.

Theorem. (Power Rule for rational exponents) Let r be any rational number.
Then
Dx (xr ) = rxr−1 .
Incomplete Proof. Since r is a rational number (i.e., a “ratio” of two integers),
we may write
p
r= ,
q
for some integers p, q, where q 6= 0. By definition, y = xp/q is a solution to the
equation

y q = xp .
1 That is, any function that has not been explicitly designed to cause problems.

3
Applying implicit differentiation, together with the power rule for integer expo-
nents, we see that

dy
qy q−1 = pxp−1
dx
dy pxp−1
= q−1
dx qy
p xp−1
= ·
q (xr )q−1
= r · x(p−1)−r(q−1)
= r · xp−1−(p/q)(q−1)
= r · xp−1−p+p/q
= r · x−1+p/q
= r · xr−1 .

The key point of this proof is that we could apply the power rule to xp and
q
y , because we already knew the power rule for integer exponents, and p, q are
integers. This proof is incomplete in that we have not really turned implicit
differentiation into a rigorous technique, so we can’t use it in “real” proofs.
I commented at one point that calculus is “supposed” to work exactly the
same for rational and irrational numbers. Thus, it seems peculiar that we have
a rule that only seems to work for rational numbers. In fact, as it turns out,
the Power Rule does hold for all real exponents—rational or irrational. There’s
even a nice, elegant proof that does not care whether r is rational or irrational.
Unfortunately, this proof uses logarithms, so we won’t see it for some time (if
at all). Thus, for now, all our powers will be rational.

2 Some potential pitfalls: numbers, functions,


and expressions
When I first introduced functions, I made a big deal of the fact that f is a
function, but f (x) is just a number (albeit one that we do not yet know). In
terms of this distinction, differentiation is something we do to functions, not
numbers. Thus, Df , the “derivative of f ,” is a function, but Df (x) would be
the “derivative of a number,” which does not make any sense. Unfortunately,
this distinction has become somewhat blurred when we write things like
d 2
(x + 1).
dx
What we really mean here is “the derivative of the function that maps x 7→
x2 + 1.” The x in d/dx tells us that x is just a “dummy variable,” and so the
input is really just a function. When we write the answer as 2x, it is even harder

4
to tell that we mean “the function mapping x 7→ 2x” rather than simply “the
number 2x.”
So far, this section has been entirely theoretical, but there is a practical,
computational issue as well. Suppose someone asks you to calculate the deriva-
tive of x2 + 1 at x = 2. You may be tempted to substitute in x = 2 before
differentiating, which would be a disaster. You’d be differentiating a number
rather than a function; you’d probably try to treat it as the constant function
22 + 1 = 5, and end up getting derivative 0 since the derivative of any constant
function is zero.
To be honest, I hope that none of you would make this particular error,
because this example is fairly straightforward. But when you deal with more
complicated relations—say, u and v are both functions of t, y is a function of
u, and you have some equation that involves all four letters t, u, v, y—it can
be easy to lose track of whether you are dealing with functions or numbers
“underneath.” A good rule of thumb here is the following:
Rule of Thumb. First, do all your differentiating. Then, and only then, start
treating variables as numbers.
For instance, if you are asked to find the derivative of x2 + 1 at x = 2, you
should first differentiate (obtaining 2x) and then substitute in x = 2 (obtaining
4, the correct answer). Like any rule of thumb, this one has occasional excep-
tions. The only truly reliable way to stay out of trouble is to know what you
are doing: to know, at each step of your argument, whether x2 + 1 really means
“the number x2 + 1” or “the function that maps x 7→ x2 + 1.” However, trying
to keep track of this can be quite confusing, and I think the Rule of Thumb
above will probably serve you well.

5
Assignment 21 (due Wednesday, 23 November)
[Note: If you won’t be in class on the day before Thanksgiving, then some time
before class, put the homework in my mailbox (in the Eckhart basement). Also,
send me an email so that I know you have done this.]

Section 2.6, Problems 11–12 and 20–21. Problems 12 and 21 will be graded
carefully.

Section 2.7, Problems 3–6 and 19–20. The even-numbered problems will be
graded carefully.

(“Semi-bonus problem”) Suppose that

lim f (x) = ` = lim+ f (x);


x→c− x→c

i.e., both the one-sided limits are defined, and they are equal. Use the ε-δ
definition of the limit to show that the two-sided limit is also defined and equal
to `, i.e., that
lim f (x) = `.
x→c

The technique involved should be similar to that used to give ε-δ proofs for
piecewise linear functions.
This is a “semi-bonus problem” in the following sense:
• If you do not seriously attempt it, you will not receive full credit on your
homework.
• If you seriously attempt it, you will receive full credit for it (although your
actual homework grade will, of course, depend on the other homework
problems).
• If you get it right, you will receive a bonus point on the homework.

Assignment 22 (due Monday, 28 November)


Section 2.5, Problems 19 and 20.

Section 2.6, Problems 23 and 38. Both of these will be graded carefully.

Section 2.7, Problems 8, 21, and 37. Problems 21 and 37 will be graded carefully.

Section 2.8, Problem 1.

6
Math 131, Lecture 24: Related Rates
Charles Staats
Wednesday, 23 November 2011

As far as I can tell, “related rates” are the textbook’s first excuse to really
start in on so-called “word problems.” Up to now, the course has been mostly
theoretical; the only real “applications” have been to studying the graphs of
functions. However, calculus was invented for real-world problems. If you can’t
understand how calculus relates to the real world, then you don’t really under-
stand calculus at all.
I think the real meat of the notion of “related rates” is in the examples, so
let us proceed to these examples without further ado.
Example 1. Suppose that a straight railroad consists of two completely rigid
segments, each 50 kilometers long. Suppose, further, that two immensely strong
men move the ends of the railroad toward each other at a constant rate of one
centimeter per hour, forcing the railroad to rise up in the center. (By this, I
mean that each end of the railroad is moving at a rate of one centimeter per
hour.) After one hour, how fast is the center point of the railroad moving up?
Solution. In any word problem like this, the first step is almost always to draw a
picture. At the same time, we probably want to assign names to all the variable
quantities.

dh/dt

50 km
50 km
h

1 cm/hr 1 cm/hr
` `

What we are interested in calculating is the rate of change of the height h with
respect to time t. We need to fix units, so let’s say we take time in hours and
distance in kilometers. We are given that the horizontal length ` is shrinking at
a rate of
cm km
1 = .00001 .
hr hr
In other words,
d`
= −.00001.
dt

1
The Pythagorean Theorem tells us that

h2 + `2 = 502 ;

differentiating both sides with respect to t, we see that

dh d`
2h + 2` = 0
dt dt
dh
2h + 2`(−.00001) = 0
dt
dh .00001`
=
dt h
1
= .
100000h
Up to now, we have only been making substitutions when we knew that
something held for all time (or at least, all t > 0). This is because we needed
to be able to differentiate; and as discussed last time, this means we are really
working with functions rather than numbers. Our variables that could change
over the course of time, would need to remain “dummy variables” so that we
could differentiate them (or with respect to them).
However, we are done differentiating now, so we can substitute in the par-
ticular case we care about: specifically, when t = 1 (i.e., after one hour). In this
case, we have that
` = 50 − .00001 = 49.99999.
Since there is an h in the formula for dh/dt, we also need to find out what h is
at t = 1, which we do using the Pythagorean Theorem (again):

h2 + `2 = 502
h2 = 502 − `2
= 502 − (50 − .00001)2
= 502 − 502 + 2 · 50(.00001) − .0000000001
= .0001 − .0000000001

h = .0001 − .0000000001
p
= 10−4 − 10−10
p
= 10−4 (1 − 10−6 )
p
= 10−2 1 − 10−6 .

2
Thus, plugging in this h, we find that

dh 1
=
dt t=1 105 h
1
= √
105
· 10−2
1 − 10−6
−5+2
10
=√
1 − 10−6
.001
=√ .
1 − 10−6

Since 1 − 10−6 is very nearly 1, this tells us that the rate of the vertex going
up, dh/dt, is very close to .001 kilometers per hour, or 1 meter per hour, at time
t = 1 hr. Thus, the midpoint is going up much faster the sides are going in (1
cm/hr).

If you really think about it, the problem above does not so much calculate
the rate of change, as explain why the problem is so incredibly unrealistic. The
way to make work easier is to use leverage, or “mechanical advantage,” so that
your quick motion produces a slow motion in the thing you are trying to move.
The fictional “very strong men” in this example are doing exactly the opposite:
they are working at an enormous mechanical disadvantage.

Assignment 22 (due Monday, 28 November)


Section 2.5, Problems 19 and 20.

Section 2.6, Problems 23 and 38. Both of these will be graded carefully.

Section 2.7, Problems 8, 21, and 37. Problems 21 and 37 will be graded carefully.

Section 2.8, Problem 1.

3
Math 131, Lecture 25
Charles Staats
Monday, 28 November 2011

1 Related Rates examples


First, let’s go over the related rates homework problem due today.
Example 1. (Section 2.8, Problem 1) Each edge of a variable cube is increasing
at a rate of 3 inches per second. How fast is the volume of the cube increasing
when an edge is 12 inches long?

Solution. Let e denote the edge length of the cube, and let V denote its
volume.

e
e
e

These two quantities are related by the equation


V = e3 .

Differentiating implicitly, we see that

dV de
= 3e2
dt dt
= 3e2 · 3
= 9e2 .
(We substituted in de/dt = 3 since this is true for all time.) At the particular
instant we care about, we are given that e = 12, and so
dV
= 9e2 = 9(12)2 = 9 · 144 = 1296.
dt

1
The volume is increasing at a rate of 1296 cubic inches per second.

Now, another example:


Example 2. (Example 1, p. 135 in the textbook) A small balloon is released at
a point 150 feet away from an observer, who is on level ground. If the balloon
goes straight up at a rate of 8 feet per second, how fast is the distance from the
observer to the balloon increasing when the balloon is 50 feet high?
Solution. I’m not going to type out the solution since it is explained in the text,
but I will leave some space here for you to take notes on what is said in class
(if you choose to do so).

2
2 Maxima and Minima
Consider a child selling lemonade on the sidewalk.1 If she sets the price at $0
per cup (i.e., she gives it away for free), then plenty of people will take a cup,
but she won’t make any money. On the other hand, if she sets the price too
high—say, $7 per cup—then no one will buy from her, and she also won’t make
any money. If she puts the price somewhere in the middle, then she may well
sell some lemonade and make some money. But how can she figure out what
price to set so that she will make the most money? Realistically, she probably
can’t—but only because she does not know calculus.2
Let m denote the amount of money she makes, let p denote the price she
charges, and let n denote the number of cups she sells. It is fairly clear that

m = n · p;

in words, the amount of money she makes is the number of cups she sells times
the price per cup.3 Moreover, we are assuming that the number of cups she
1 Let’s pretend it’s summer; otherwise, she has chosen a singularly inappropriate time of

year for her enterprise.


2 Okay, I’m exaggerating here. She would also need to have done a fair amount of market

research, and even then the answer would only be approximate. But since this is a course in
calculus rather than economics, we’re going to ignore that bit.
3 You might object that we should also consider how much she has to pay for the lemonade,

but I’m assuming her mom covers that for her.

3
sells is determined by the price she sets. In other words, n is a function of p.
Consequently, m is also a function of p.
Ideally, we should do a fair amount of market research to figure out what
function gives n; in other words, how many cups sells when she sets a given
price. But since we’re mathematicians rather than economists here, let’s just
make a sort of silly guess. Let’s say that if she sets the price at $0, then she will
“sell” (give away) 50 cups (maybe 50 people pass by during the hour she sits at
the stand). If she sets the price to $7 or more, she will sell zero cups. So, let’s
just draw a straight line between the points (0, 50) and (7, 0), and call it n.
n

50

p
$7

The corresponding function is


(
50 − 50
7 p if 0 ≤ p ≤ 7,
n=
0 if p > 7
( 
50 − 50
7 p p if 0 ≤ p ≤ 7,
m = np =
0·p if p > 7
(
50p − 50
7 p
2
if 0 ≤ p ≤ 7,
=
0 if p > 7.

Note that these functions are not defined for p < 0, since “negative price” really
does not make sense in this context.
If we graph m, the amount of money made, as a function of the price p, we
obtain

4
m

$90
$80
$70
$60
$50
$40
$30
$20
$10
$0 p
$0 $1 $2 $3 $4 $5 $6 $7 $8

The maximum value is the point where the tangent line to the graph is horizontal—
in other words, where m0 (p) = 0. And we can find this using calculus:
(
50p − 50
7 p
2
if 0 ≤ p ≤ 7,
m(p) =
0 if p > 7.
(
50 − 100
7 p if 0 < p < 7,
m0 (p) =
0 if p > 7.

Warning. One error that a lot of people made on the test would amount, in
this case, to writing m0 (p) = 50 − 100
7 p for 0 ≤ p ≤ 7. (Note the ≤ sign rather
than the < sign.) When you differentiate a piecewise-defined function, a ≤ sign
will usually (although not always) become a < sign. If you look at the graph,
you can see that the function is not differentiable at p = 7.
If we solve for the places where m0 (p) = 0, we find that this holds when
p = 3/2 or p > 7. Looking at the graph, it is clear that m is maximized
(i.e., the girl makes the most possible money) when p = 7/2 = 3.5; in other
words, according to this model, she ought to set her price at $3.50 per cup. The
maximum value of the function is
  
7 2
m 72 = 50 72 − 50 7 2 = 87.5.

In other words, the most money the girl can possibly make is $87.5.
The following, more precise mathematics allows us to handle these sorts of
things more generally:
Definition. Let f be a function defined on an interval [a, b] and x0 a point in
its domain. We say that x0 is a critical point of f if any of the following holds:
• x0 is an endpoint of the interval (i.e., x0 = a or x0 = b); or
• f 0 (x0 ) does not exist; or

5
• f 0 (x0 ) = 0.
The last type of critical point, where f 0 (x0 ) = 0, is in some sense the most
interesting sort of critical point to find (find the derivative f 0 , then solve for
f 0 (x) = 0). But the other two kinds should not be forgotten, since they are
absolutely necessary to make the following theorem true.

Theorem. Let f be a continuous function with domain a closed interval [a, b].
Then f has a maximum value and a minimum value. Moreover, every point at
which the maximum (minimum) is attained is a critical point.
In other words, if we know f is a continuous function on [a, b], then the
following procedure will allow us to find the minima and maxima of f on [a, b]:
1. Find the critical points of f (all three kinds).
2. Evaluate f at each of the critical points.
3. The largest of the resulting values is the maximum value of f on [a, b].
The least of the resulting values is the minimum value of f on [a, b].

6
Assignment 23 (last assignment; due Wednesday,
30 November, 2011)
Section 2.7, Problems 21, 22, and 38.

Section 2.8, Problems 3, 6, and 17(a,b).

Section 3.1, Problems 1 and 5–6. On 5 and 6, include graphs of the function on
the interval. Do NOT graph the function outside the interval.
Bonus Exercise. Let f be a function defined on (0, 1); in other words, f (x) is
defined whenever 0 < x < 1.
(i) Give a formal (M -δ) definition for the statement that

lim f (x) = ∞.
x→0+

(ii) Assume that lim+ f (x) = ∞. Use the M -δ statement above to prove that
x→0
f has no maximimum value on (0, 1).

7
Math 131, Lecture 26 (final lecture)

Charles Staats
Wednesday, 30 November 2011

1 Logistics: Review session on Friday


Since reading period starts tomorrow, class on Friday will not introduce any
new material. Instead, I will devote the class to answering students’ questions.
I expect most of the time to be spent on going over how to solve different kinds
of problems, but I will also take questions about what sorts of things are and
are not fair game for the exam.
Attendance is not required, but I think the class will be more helpful for
everyone if a lot of people show up. I want people to do well on the final, and
it is very frustrating for me when people miss something that they could have
gotten right if they had only asked me to explain it.
Chaofan and Jay will not be holding tutorials tomorrow. Seth, however, will
(in Pick 022); everyone is welcome to attend, whether or not you are in Seth’s
tutorial normally.

2 Maxima and minima—motivation applications


We will be studying how to use calculus (specifically, derivatives) to find points
at which a function is maximized or minimized. This sort of thing has many
practical applications. For instance, we can ask
• What price should we sell lemonade at in order to make the most profit?
(Profit is a function of price; we want to select price to maximize it.)
• What shape should a rectangle be to fence in the largest possible area
with a fixed amount of fence? (The area of the rectangle is a function of
its length; we want to maximize it.)
• What path should a pipeline follow under a river to minimize the cost of
building it? (The cost is a function of the path; we want to minimize this
function.)
We won’t get to solve these sorts of problems in this lecture (and thus not until
next quarter), but I will show you the mathematical tools that are used to solve
them.

1
3 Maxima and minima—the theory
We’re going to spend a few minutes talking about the basic theory (theorems
and such) before seeing the applications.
Definition. Let f be a function. The maximum value of f is a value M such
that
(i) f attains the value M ; i.e., there is some x0 such that M = f (x0 ); and

(ii) M ≥ f (x) for all x in the domain of f .


The minimum value of f is a value m such that
(i) f attains the value m; i.e., there is some x0 such that m = f (x0 ); and

(ii) m ≤ f (x) for all x in the domain of f .


An extreme value of f is a value y that is either the maximum or the minimum
value of f .
Warning. Maximum and minimum values need not exist; consider the following
two cases.

2
f (x)

g(x)

(
x if 0 ≤ x < 1,
g(x) = 1
2 if 1 ≤ x ≤ 2.

The maximum of this function


x
“should” be 1, but in fact the function
has no maximum because it never
quite reaches 1. There is no point x0
1 such that g(x0 ) = 1.
f (x) = if 0 < x ≤ 1
x
This function has no maximum be-
cause it attains arbitrarily large val-
ues.
In both of the cases above, the “issue” was that there were points at which
the function had no finite limit. Specifically,

lim f (x) = ∞, while lim g(x) does not exist.


x→0 x→1

This yields plausibility to the following theorem:

Theorem. Let f be a continuous function on a closed interval [a, b]. Then f


has a minimum and a maximum.
We won’t even try to prove this. For the function f above, the function
was defined on (0, 1], but 0 was missing from the domain—the interval was not
closed. For g, the function was not continuous.
The points where minimum and maximum values might take place are called
critical points. More precisely,
Definition. Let f be a function defined on an interval [a, b] and x0 a point in
its domain. We say that x0 is a critical point of f if any of the following holds:

3
• x0 is an endpoint of the interval (i.e., x0 = a or x0 = b); or
• f 0 (x0 ) does not exist; or
• f 0 (x0 ) = 0.

The last type of critical point, where f 0 (x0 ) = 0, is in some sense the most
interesting sort of critical point to find (find the derivative f 0 , then solve for
f 0 (x) = 0). But the other two kinds should not be forgotten, since they are
absolutely necessary to make the following theorem true.
Theorem. Let f be a continuous function with domain a closed interval [a, b].
Then the only points where f could possibly equal its extreme values are the
critical points.
Idea of proof. We prove the contrapositive. Suppose x0 is not a critical point.
We will show that f (x0 ) is not an extremal value of f .

f (x)

f (x0 )

x
x0

Since x0 is not a critical point, x0 is differentiable and f 0 (x0 ) 6= 0. In other


words, f has a tangent line at x0 that is not horizontal. Thus, for x sufficiently
close to x0 , f (x) is contained in a narrow cone about the tangent line.
Since the tangent line is not horizontal, if we make the cone sufficiently
narrow, we can ensure that the values of f immediately to the right of x0 (if
the slope is positive) or immediately to the left of x0 (if the slope is negative)
are above f (x0 ). Since x0 is not a critical point, it is not an endpoint of the
domain, so f does have values immediately to the left and right of x0 . Hence,
f (x0 ) is not an maximum of f .
Similar reasoning shows that f (x0 ) is not a minimum value of f .

4
4 Maxima and minima: example
In other words, if we know f is a continuous function on [a, b], then the following
procedure will allow us to find the minima and maxima of f on [a, b]:
1. Find the critical points of f (all three kinds).
2. Evaluate f at each of the critical points.

3. The largest of the resulting values is the maximum value of f on [a, b].
The least of the resulting values is the minimum value of f on [a, b].
Example 1. Find the critical points, minimum, and maximum for the function
f given by
f (x) = 13 x3 − x
on the closed interval [−2.5, 1.5].

f (x)

x
−4 −3 −2 −1 1 2 3 4
−1

−2

−3

−4

You might also like

pFad - Phonifier reborn

Pfad - The Proxy pFad of © 2024 Garber Painting. All rights reserved.

Note: This service is not intended for secure transactions such as banking, social media, email, or purchasing. Use at your own risk. We assume no liability whatsoever for broken pages.


Alternative Proxies:

Alternative Proxy

pFad Proxy

pFad v3 Proxy

pFad v4 Proxy